Сохранен 683
https://2ch.hk/spc/res/223277.html
Домены arhivach.top и arhivach.site временно не функционируют! Используйте домен ARHIVACH.XYZ.
24 декабря Архивач восстановлен после серьёзной аварии. К сожалению, значительная часть сохранённых изображений и видео была потеряна. Подробности случившегося. Мы призываем всех неравнодушных помочь нам с восстановлением утраченного контента!

Тред тупых вопросов. #36

 Аноним 25/10/15 Вск 19:46:49 #1 №223277 
14457916095010.jpg
14457916095011.webm
14457916096282.webm
Тред вопросов о жизни, Вселенной и всем таком.

Спрашиваем то, за что в других местах выдают путёвку в биореактор. Здесь анонимные ученые мирового уровня критически рассмотрят любые гениальные идеи и нарисованные в Paint схемы.

Прошлый тред https://2ch.hk/spc/res/219994.html
Аноним 25/10/15 Вск 20:00:40 #2 №223279 
14457924401190.jpg
Можно ли просветить планету насквозь чем-то, кроме нейтрино? Можно ли таким образом узнать о её внутреннем строении?
Аноним 25/10/15 Вск 20:01:50 #3 №223280 
>>223279
Сейсмология (долбануть посильнее), грав. волны, в перспективе.
Аноним 25/10/15 Вск 20:11:53 #4 №223282 
>>223280
так там отражение же, а мне интересно именно насквозь. Допустим вот газовый гигант какой-нибудь с маленьким ядром и мощной атмосферой. Можно ли просветить гиганта, так чтобы увидеть это ядро плавающим где-то в центре этой толстенной атмосферы?
Аноним 26/10/15 Пнд 03:54:26 #5 №223340 
>>223277 (OP)
>Space Rocks
Такая милая видюшка!
Аноним 26/10/15 Пнд 11:28:54 #6 №223385 
>>222836
> Мой научный метод подсказывает, что именно вращение Солнца вокруг Земли есть причина эрекции, которую я обнаруживаю каждое утро.
А нихуя. Вот если ты обнаружишь отсутствие стояка по утрам когда Земля не будет вращаться вокруг Солнца, тогда я тебе поверю.
Аноним 26/10/15 Пнд 12:30:58 #7 №223391 
>>223277 (OP)
Что на второй вебмке? Не нашёл канал.
Аноним 26/10/15 Пнд 18:26:24 #8 №223464 
Почему Солнечная система плоская? (Я имею в виду орбиты массивных тел, не астероидов)
Почему галактики обычно плоские?
Аноним 26/10/15 Пнд 19:06:53 #9 №223482 
14458756135470.jpg
>>223464
Если по-простому - все что вращается, обладает осевой симметрией и стремится сплющиться в диск вокруг этой оси. То же и произошло с изначальными говнами, из которых сложилась и галактика, и солнечная система, и планеты.
Аноним 26/10/15 Пнд 19:10:32 #10 №223485 
>>223482
А откуда взялся первичный вращательный момент у говен, направленный именно так?
Аноним 26/10/15 Пнд 19:16:52 #11 №223489 
>>223485
Во Вселенной вообще ничего не стоит на месте, все находится в движении. Во-вторых, когда схлопывается газопылевое облако, превращаясь в протозвезду, оно начинает тащить в свою сторону пыль и газ, далее закон сохранения момента импульса
Аноним 26/10/15 Пнд 19:18:41 #12 №223493 
>>223391
>Что на второй вебмке?
Рассказывают, как не путать названия каменюк, падающих (или не падающих) на голову.
Аноним 26/10/15 Пнд 19:30:12 #13 №223499 
>>223485
Из изначальной неоднородности Вселенной, которая просто была.
Аноним 26/10/15 Пнд 19:30:16 #14 №223500 
14458770165140.jpg
Объясните про реликтовое холодное пятно. Почему о нем говорят, что это отпечаток соседней Вселенной рядом? И как вообще рядом может быть другая Вселенная, если Вселенная это все сущее?
Аноним 26/10/15 Пнд 19:34:48 #15 №223502 
>>223489
>>223499
Видимо, раз вся Вселенная совокупно не похожа на блин, то у Вселенной как единого целого вращательного момента нет?
Аноним 26/10/15 Пнд 19:36:03 #16 №223503 
>>223500
>Почему о нем говорят, что это отпечаток соседней Вселенной рядом?
Забудь об этом. Это какой-то долбоеб притащил на википедию абзац этого бреда, и, видимо, жопой защищает его от удаления.
Аноним 26/10/15 Пнд 19:37:50 #17 №223505 
>>223502
>Видимо, раз вся Вселенная совокупно не похожа на блин
С чего ты решил что не похожа?
Аноним 26/10/15 Пнд 19:40:53 #18 №223508 
>>223502
А на масштабах Вселенной это не работает, т.к. там разбегание пересиливает гравитацию.
Аноним 26/10/15 Пнд 19:42:35 #19 №223509 
>>223500
В прошлом треде поясняли.
>Почему-то на русскоязычной вики приведена только одна гипотеза возникновения этого пятна, и при том самая непроверяемая.
Самым простым и логичным объяснением можно считать, что это холодное пятно ни что иное, как войд очень больших размеров - огромное практически полностью пустое пространство.
Аноним 26/10/15 Пнд 19:44:10 #20 №223510 
>>223502
>Видимо, раз вся Вселенная совокупно не похожа на блин
Нам ничего не известно о топологии и форме Вселенной на больших маштабах. Она может быть похожей на двух ебущихся ежиков, а может быть не похожей.
Аноним 26/10/15 Пнд 19:52:46 #21 №223512 
>>223277 (OP)
Вебм с обработанными фотками просто до мурашек. Потрясающе красиво. Схоронил
Аноним 26/10/15 Пнд 21:35:20 #22 №223526 
>>223509
Насколько огромное? (предположительно)
Аноним 26/10/15 Пнд 21:57:30 #23 №223527 
>>223526
>Насколько огромное? (предположительно)
Около миллиарда световых лет.
Аноним 26/10/15 Пнд 22:00:53 #24 №223529 
>>223526
1,8 миллиарда св лет.

В принципе суть пятна не в том, что оно сильно холодное, статистически оно не холоднее других холодных мест, и не больше их, просто на фоне окружающих его горячих областей оно выделяется.
Аноним 26/10/15 Пнд 23:22:43 #25 №223537 
>>223489
Не совсем. Осевая симметрия в рассматриваемом нами случае — следствие гравитации Солнца. Говоря грубо, планеты будто бы "пытаются" упасть на него, но ничего не выходит ввиду ненулевого импульса.
Аноним 27/10/15 Втр 00:07:59 #26 №223541 
>>223537
Прежде чем открыть рот, и начать вещать что-то умное на твой взгляд, потрудись почитать, о чем речь идет.
Аноним 27/10/15 Втр 00:14:46 #27 №223543 
>>223541
Ты ему попытался объяснить чуть осевой симметрии. Я немного раскрыл тему. Вопрос: почему тебе теперь больно?
Всё из-за баннеров.
Аноним 27/10/15 Втр 00:51:10 #28 №223554 
>>223543
Потому что ты нихуя не расскрыл, он спрашивал, почему планеты "блином" летают, в одной плоскости а ты начал пиздеть очевидное про гравитацию. Какую тему ты раскрыл? Какие нахуй планеты, если еще до их формирования исходное говно крутилось в плоском диске?

Ты по баннерам сюда пришел что ли?
Аноним 27/10/15 Втр 00:55:41 #29 №223555 
>>223489
>когда схлопывается газопылевое облако, превращаясь в протозвезду, оно начинает тащить в свою сторону пыль и газ, далее закон сохранения момента импульса
Это если повезет что облако имеет собственный момент импульса. Если не повезет то получится одинокая не вращающаяся звездочка.
>>223464
>Почему Солнечная система плоская?
Очевидно в молодости, когда солнышко еще не вращалось, оно попало в галактический бар, а это мощный поток материи, способный закрутить любую звезду. Так вот, по мере раскрутки от нее отделялись один за одним куски, это будущие ураны-юпитеры, эти куски тоже раскручивались и от них отделялись кусочки поменьше - всякие титаны, европы, тритоны. Потом, выйдя из бара, поток начал ослабевать, тогда вылезли марсы-меркурии. С землей правда случился казус, каким-то образом ядро солнышка родило сразу два комка, они вылезли и начали вращацо совместно.
Аноним 27/10/15 Втр 01:25:20 #30 №223557 
Уважаемые Аноны отсыпьте, нужны документалки, только не СНГшные, у самого руки кривые, всё что нашел просмотрел, но знаю что есть ещё, далекий космос манит, а топлива нет для продолжения пути в прекрасное ничто, прошу помощи.
Аноним 27/10/15 Втр 03:05:17 #31 №223563 
>>223555
>Это если повезет что облако имеет собственный момент импульса.
Он не нужен, достаточно неоднородности. Если облако большое и неоднородное - момент будет большой. Если нет - маленький.
Аноним 27/10/15 Втр 03:35:20 #32 №223564 
>>223555
>то получится одинокая не вращающаяся звездочка.
Не получится. Когда облако начнет коллаписировать и стягивать газ оно придаст движение и вращение вокруг первоначальной плотной неоднородности, которая в итоге станет протозвездой. Вращение всегда будет.

>Так вот, по мере раскрутки от нее отделялись один за одним куски, это будущие ураны-юпитеры
Твои познания в звездной эволюции и формирования планетраных систем равно нулю. Сходи почитай что-нибудь, для начала.
Аноним 27/10/15 Втр 03:43:20 #33 №223565 
>>223557
https://2ch.hk/spc/res/201142.html

От себя рекомендую серию документалок с Брайаном Коксом. А вообще книжки читай, научпопс - это развлекалово, а не источник знаний.
Аноним 27/10/15 Втр 05:23:13 #34 №223569 
14459125934720.jpg
Насколько большую опасность для МКС, спутников в космосе, представляют собой микрометеориты?
Было много случаев столкновения в космосе со всяким хламом?
Астронавты на МКС сильно рискуют?
Аноним 27/10/15 Втр 06:11:19 #35 №223571 
14459154793320.jpg
14459154793321.jpg
>>223569
>Насколько большую опасность для МКС
Зависит от размера, скорости и траектории. Но за все время, что проводятся полеты не было ни одного серьезного случая.

>Было много случаев столкновения в космосе со всяким хламом?
Бывало, но не много. Больше проблем доставляет космический мусор, которым насрали сами люди, чем непосредственно микрометеориты.

>Астронавты на МКС сильно рискуют?
Риск откинуть копыта от микрометеорита довольно сомнителен, люди похерели в космических программах около 20 человек, а микрометеориты не стали причиной ни одного случая с угрозой жизни.
Аноним 27/10/15 Втр 06:47:12 #36 №223573 
14459176328040.jpg
А есть где-нибудь сайты с детальным разбором падений ракет и фейлов АМС, спутников и т.п.? Что сломалось, почему, как боролись за живучесть аппарата, какие выводы были сделаны?
Аноним 27/10/15 Втр 06:49:43 #37 №223574 
>>223573
Ну или почитать что-нибудь по этой теме.
Аноним 27/10/15 Втр 08:29:58 #38 №223575 
14459237986760.jpg
>>223537
>"пытаются" упасть на него
Ок, вот уже третий год протираю в спейсаче штаны и меня в конец утомило это объяснение для даунов. Настало время узнать правду. Возможно даун на самом деле я.
Что блять значит ваше "пытается упасть, но постоянно промахивается"? В случае с орбитальным движением мы имеем нулевую результирующую, сила тяжести уравновешена центробежной. Что значит пытается, и как понять "промахивается"?
Аноним 27/10/15 Втр 08:31:20 #39 №223576 
>>223555
>Это если повезет что облако имеет собственный момент импульса. Если не повезет то получится одинокая не вращающаяся звездочка.
Ват? Много нашли звезд, не имеющих осевого вращения?
Аноним 27/10/15 Втр 08:59:42 #40 №223577 
14459255822960.jpg
>>223575
>Что блять значит ваше "пытается упасть, но постоянно промахивается"
Наше "пытается упасть, но постоянно промахивается" блядь означает, что мы тут в треде тупых вопросов, подразумевающим понятные любому дегенерату ответы.
Аноним 27/10/15 Втр 11:49:41 #41 №223585 
>>223563
А какие книжечки посоветуешь ?
Аноним 27/10/15 Втр 11:50:45 #42 №223586 
>>223565
а какие книжечки посоветуешь ?
Аноним 27/10/15 Втр 12:47:14 #43 №223597 
>>223563
>Он не нужен, достаточно неоднородности.
Попробуй смоделируй. Без начального момента ничего не выйдет. Все формулы небесной механики основаны на законах сохранения, это значит что лишнему моменту неоткуда взяться если его в начале не было.
Аноним 27/10/15 Втр 12:50:06 #44 №223598 
>>223564
>Вращение всегда будет.
Да, небольшое вращение будет, но вот на образование планет момента уже не хватит.
Аноним 27/10/15 Втр 13:20:43 #45 №223601 
>>223554
Газопылевые облака из твоего высказывания могут в том числе образовывать звёзды.
Вот, что ты выдал:
>когда схлопывается газопылевое облако, превращаясь в протозвезду, оно начинает тащить в свою сторону пыль и газ, далее закон сохранения момента импульса
Ты неумело попытался раскрыть тему макро-вращения в треде тупых вопросов и упомянул только закон сохранения момента импульса.

Залётный из /b пытается проецировать, потешно! Когда вы передохните?
Ты, наверное, из KSP? Честно, я давно не видел таких тугодумных.
Аноним 27/10/15 Втр 17:21:33 #46 №223651 
14459556936800.jpg
Посоны, наблюдаю пикрелейтед с утра примерно в юго-восточной стороне (градусов примерно 60 относительно горизонта). Бульбостан-кун

Это же Венера и Марс?
Аноним 27/10/15 Втр 17:33:26 #47 №223652 
14459564065520.png
>>223651
Стеллариум себе поставь.
Аноним 27/10/15 Втр 21:08:39 #48 №223705 
>>223651
Первая попавшаяся газетенка:
http://www.dni.ru/tech/2015/10/27/318867.html
Аноним 28/10/15 Срд 05:43:38 #49 №223777 
>>223277 (OP)
Поясни анон такую штуку - все экзопланеты которые в последнее время обнаруживаю находятся в нашей галактике, правильно? А что на счет планет ЗА пределами нашей галактики?
Аноним 28/10/15 Срд 06:10:27 #50 №223779 
>>223777
>Поясни анон такую штуку - все экзопланеты которые в последнее время обнаруживаю находятся в нашей галактике, правильно? А что на счет планет ЗА пределами нашей галактики?
Не видны ни в каком диапазоне волн. Далеко слишком.
Аноним 28/10/15 Срд 06:34:52 #51 №223781 
Под поверхность Марса возможна жидкая вода, подземные воды подобные земным?
Аноним 28/10/15 Срд 06:37:23 #52 №223783 
>>223781
Да, возможны, причем в свете последних наблюдений,раз уж на поверхности подтекает, то может и внутри булькает. Если Марс не промерз на несколько км в глубь.
Аноним 28/10/15 Срд 06:54:01 #53 №223784 
>>223779
Спасибо. Мы обречены "знакомству" только с планетами и звездами нашей сраной галактики. Но ведь в других галактиках должны быть планеты? А следовательно жизнь, так?
Аноним 28/10/15 Срд 07:38:43 #54 №223785 
>>223784
Да, мы "обречены", а теперь съеби. Мало ему планет блять.
Аноним 28/10/15 Срд 08:08:50 #55 №223786 
>>223785
Нетъ. Не съебу.
Аноним 28/10/15 Срд 08:58:31 #56 №223788 
>>223786
В Галактике (с большой буквы) миллиарды звезд. Это, конечно, заебись - мечтать о жизни в другой галактике, но у нас тут пока кучи говна не перелопачены, в собственной галактике.

Так что читай лучше учебники по физике, вместо ололо, никанон звездных войн.
Аноним 28/10/15 Срд 18:40:48 #57 №223867 
Система тоже летит или солнце стоит на месте?
Аноним 28/10/15 Срд 18:47:10 #58 №223868 
>>223571
У спутника уже скорость огромная, так что столкновение даже с гвоздём прошьёт его насквозь. Очень странно, что никого даже не ранило до сих пор.
Аноним 28/10/15 Срд 18:47:21 #59 №223869 
>>223867
Система тоже летит.
Аноним 28/10/15 Срд 18:47:27 #60 №223870 
>>223788
Всё равно до них никогда не добраться.
[вфывфыв] Аноним 28/10/15 Срд 19:03:58 #61 №223873 
>>223277 (OP)
анон поясни мне как пользоваться подвижной картой звездного неба?
в гуголе ничо нипанятна.
Аноним 28/10/15 Срд 19:06:09 #62 №223875 
>>223873
Двигаешь и пользуешься.
[вфывфыв] Аноним 28/10/15 Срд 19:24:35 #63 №223881 
>>223875
>>223875
а как по времени орентироваться??? если с другой стороны совсем оно например
Аноним 28/10/15 Срд 21:19:30 #64 №223905 
>>223482
>все что вращается, обладает осевой симметрией и стремится сплющиться в диск вокруг этой оси
Тогда и атом должен быть плоским? То есть правильно говорить не "электронное облако", а "электронный диск"?
Аноним 28/10/15 Срд 21:20:36 #65 №223906 
>>223905
Электронная лепёшка.
Аноним 28/10/15 Срд 23:01:17 #66 №223930 
почему так мало ракет стартует с самолётов? этож халявные тышша-две км от поверхности плюс некоторый импульс, пусть и направленный не совсем туда куда надо.

и сразу следующий вопрос: почему нет маленьких кораблей которые использовали бы атмосферу в качетсве среды для разгона? человек многократных перегрузок конечно не выдержит, но железки или скажем бронепластины таким образом на орбиту можно было бы засылать, не?
Аноним 29/10/15 Чтв 00:12:28 #67 №223939 
>>223930
> среды для разгона?
Для разгона как раз среда-то и нинужна
Аноним 29/10/15 Чтв 01:18:27 #68 №223948 
>>223930
В дозвуковом воздушном старте слишком много тонкостей ради небольшой экономии, в результате по стоимости это себя не оправдывает. К тому же для серьезных нагрузок нужен ёба-самолет, который тоже не построишь за бесплатно. Из плюсов - ненужность сложного стартового стола и возможность стартовать из любой точки шарика на любое наклонение, что несколько упрощает последующие маневры РБ, но эти плюсы не настолько востребованы, чтобы оправдать минусы.

> почему нет маленьких кораблей которые использовали бы атмосферу в качетсве среды для разгона?
Ты имеешь в виду воздушный старт либо SSTO с горизонтальным взлетом? Потому что ТРД тянут всего лишь до сраных 3.5 махов, и то любой сверхзвук, а тем более такой, очень дорогой.

Чтобы был какой-то смысл, нужен дешевый и экономичный гиперзвуковой разгонщик. А такой вряд ли будет в обозримом будущем, т.к. гиперзвуковой полет гораздо сложней чем космический, и изучен слабо.
Аноним 29/10/15 Чтв 01:19:15 #69 №223949 
>>223948
И да, стартового стола-то может и нет, но предстартовая подготовка не сильно проще все равно.
Аноним 29/10/15 Чтв 01:35:30 #70 №223952 
>>223949
Стартовый стол это сам самолет, и все кроме бетоного основания все равно придется возить с собой. А для ракет класса "пегаса" и наземные стартовые комплексы попроще будут.
Алсо, про пуски на любые наклонения тоже сорт оф хуйня: дальность у самолета ограничена, потому сильно далеко не улетишь. А еще южные соседи будут ругаться на отработанные ступени.
Аноним 29/10/15 Чтв 03:36:21 #71 №223970 
>>223868
Космос в основном пуст, при совершенно неебических размерах.
Аноним 29/10/15 Чтв 03:41:45 #72 №223971 
>>223867
Солнце делает оборот вокруг Галактики за 220-250 млн лет. При этом еще двигаясь в плоскости галактического диска вверх-вниз, примерно раз в 30 миллионов лет его пересекая.
Аноним 29/10/15 Чтв 03:43:15 #73 №223972 
>>223905
http://elementy.ru/news/430940
Аноним 29/10/15 Чтв 04:01:29 #74 №223974 
>>223905
Электроны не орбитируют, а орбиталь - не орбита, а облако вероятности обнаружения.
Аноним 29/10/15 Чтв 06:46:06 #75 №223986 
14460903660640.png
1. Если время время едино и бесповоротно, то почему во снах я вижу будущее?
2. Если вселенная одна, то почему иногда я вижу и знаю как бы жил другой я, если бы поступил не так, как в этой вселенной?
Аноним 29/10/15 Чтв 07:04:08 #76 №223988 
14460914489380.gif
>>223986
> я вижу будущее
пруфы
> я вижу и знаю как бы жил другой я
пруфы
А потом ты просыпаешься и всё забываешь, озоза!

Да ты же долбаный Квисатц Хадерах!
Аноним 29/10/15 Чтв 07:55:14 #77 №223992 
>>223986
>то почему во снах я вижу будущее?
>почему иногда я вижу и знаю как бы жил другой я
Так тебе в /mg/ надо, или /sn/, раньше можно было еще в /bb/ свои трипы описывать, но его выпилили.
Аноним 29/10/15 Чтв 11:07:12 #78 №224010 
>>223870
Почему? Если сможем разгоняться хотя бы до 0,3с - спокойно можно освоить какие-то системы из нашего рукава, разве нет? Ну конечно лететь придется на кораблях поколений.
Аноним 29/10/15 Чтв 13:35:17 #79 №224015 
>>224010
Почему поколений? У нас куча звёзд в пределах 15 св лет, на 0.3 С лет за 50 можно долететь, даже с учётом нескольких лет на разгон-торможение.
Нужно прокачать медицину и самореплицирующихся и саморемонтирующихся роботов, которые строить базы будут. И то и другое сейчас бурно развивается, так что может до старта первого межзвёздного кто-то и доживёт.
Аноним 29/10/15 Чтв 14:38:42 #80 №224024 
>>224015
Потому что в пределах 15 с.л. не открыто пока экзопланет земного типа, скорее всего расстояния будут 25-100 с.л. До ближайших двух, видимо, 22 и 36 с.г. https://ru.wikipedia.org/wiki/Список_ближайших_экзопланет_земного_типа
Аноним 29/10/15 Чтв 14:39:14 #81 №224025 
>>224015
Задач нет. Автоматические станции справятся.
Аноним 29/10/15 Чтв 14:59:45 #82 №224030 
>>224010
Остается только один вопрос - нахуя?
Аноним 29/10/15 Чтв 15:03:49 #83 №224034 
>>224030
Увидеть Париж голую каменюку и умереть?
Аноним 29/10/15 Чтв 15:09:20 #84 №224037 
>>224010
>Если сможем разгоняться хотя бы до 0,3c
Аноним 29/10/15 Чтв 15:11:12 #85 №224039 
>>224030
Расселяться. Потому что нефть кончится, солнце умрет, а обломки кометы Шумейкеров — Леви 9 однажды могут упасть на нас. Да и в целом, почему бы не засрать галактику и не устроить войну с жуками?
>>224037
Теоретически пока нет непреодолимых трудностей. Практически - пока даже не не рядом.
Аноним 29/10/15 Чтв 17:37:34 #86 №224068 
>>224039
>Практически - пока даже не не рядом c тем местом которое рядом
Аноним 29/10/15 Чтв 17:58:16 #87 №224072 
>>223277 (OP)
В чём смысл жизни?
Аноним 29/10/15 Чтв 18:12:46 #88 №224080 
>>224072
прост))
Аноним 29/10/15 Чтв 18:32:30 #89 №224082 
>>224072

42
sageАноним 29/10/15 Чтв 18:34:51 #90 №224083 
>>224039
>Расселяться
Пока, вроде, на самой земляшке преобладают процессы урбанизации, народ из ебеней стекается в города, а не наоборот. Ну а нефть не использовали еще 150-200 лет назад, и ничего, не дристали жиденьким от мысли, что она кончится.
Аноним 29/10/15 Чтв 18:42:58 #91 №224084 
>>224068
У меня вся надежда на технологическую сингулярность.
>>224083
1. Объясни это комете.
2. 150-200 лет назад мир не был так зависим от энергоносителей и соответственно население было 1 млрд, а не 7.
3. Необходимость расселяться достаточно очевидна, с точки зрения выживания вида, формулируется очень просто "не складывай все яйца в одну корзину".
Аноним 29/10/15 Чтв 18:47:02 #92 №224085 
14461336224930.jpg
>>224084
> Объясни это комете.
kek
Аноним 29/10/15 Чтв 18:50:39 #93 №224086 
>>224084
>3
Для выживания вида надо время от времени добавлять свежей крови, а не плодить жертв инцеста на далёкой планете.
Аноним 29/10/15 Чтв 18:54:24 #94 №224087 
>>224084
> 3. Необходимость расселяться достаточно очевидна, с точки зрения выживания вида, формулируется очень просто "не складывай все яйца в одну корзину".
Дали тебе Луну, Марс - нет, хочу морозить жопу на Кеплер-12432C.
Аноним 29/10/15 Чтв 18:58:45 #95 №224088 
>>224086
Ты с мамкой вдвоем собрался колонизировать или порассуждаем о минимальном количестве аллелей на локус и средней гетерозиготности?
>>224087
>Луна
Автономность под вопросом, большая популяция еще под бОльшим вопросом.
>Марс
Негостеприимный, атмосферы нет, воды мало, холодный. Тогда уж спутники гигантов, тоже холодные, зато есть с водой и есть с углеводородами.
А если найти планету - где можно будет без купола жить, пусть и не сразу... Потому и интересны экзопланеты.
Аноним 29/10/15 Чтв 19:16:37 #96 №224090 
>>224088
>воды мало
Уносите этого пленетолога.
Аноним 29/10/15 Чтв 19:25:28 #97 №224093 
>>224090
По существу есть что сказать или ты на полюсах вместе с сухим льдом ее копать собрался?
Аноним 29/10/15 Чтв 19:29:27 #98 №224094 
>>224093
По существу скажу, что на полюсах воды дохуя, а углекислота там выпадает небольшим слоем и то зимой. Что касается остального Марса, то воды дохуя в грунте.
Так что ты лучше про аллели давай.
Аноним 29/10/15 Чтв 19:30:35 #99 №224095 
>>224088
> А если найти планету - где можно будет без купола жить, пусть и не сразу...
За морем телушка - полушка, да рубль перевоз.
Твое "расселение" если и будет, то в любом случае пойдёт через Марс/Луну.
Аноним 29/10/15 Чтв 19:33:33 #100 №224096 
>>224094
А ты осознаешь, что база на полюсе неудобна по дохуя скольки причинам?
Про аллели давай, можно на корабле поколений увезти популяцию, которая не будет вырождаться.
>>224095
Чем хороша луна как перевалочная база я еще могу понять, хотя опять же, спутники гигантов еще более лучше подойдут. А вот чем марс-то хорош для дальнейшего расселения, кроме возможности до него долететь уже сейчас?
Аноним 29/10/15 Чтв 19:37:42 #101 №224097 
>>224096
> А вот чем марс-то хорош для дальнейшего расселения, кроме возможности до него долететь уже сейчас?
Тем, что в бесконечное множество раз реалистичнее еще не открытой планеты у черта на куличиках, которая в лучшем случае будет отличатья от Марса наличием нахуй не нужного магнитного поля и, если очень повезет, атмосферы из малопригодной для дыхания хуйни??
Аноним 29/10/15 Чтв 19:43:03 #102 №224098 
>>224097
Преимущество перед луной только одно: "марсианин" звучит лучше "лунатика". Жить под куполом можно и на спутниках гагантов, еще раз повторю, а там и третья космическая ближе и дохуя всего есть, кроме солнца и тепла, собственно на марсе с ним тоже не ахти. Конечно лететь до марса меньше, но все равно же пока не будет стабильных перелетов - не будет колонии, а там может и научимся не годами летать между планетками.
Аноним 29/10/15 Чтв 19:43:24 #103 №224099 
>>224098
гигантов
быстрофикс
Аноним 29/10/15 Чтв 19:47:19 #104 №224100 
>>224097
А вообще - чем плохо лететь к планетам у черта на куличках, если там есть хотя бы давление? А если есть еще и кислородосодержащая атмосфера - можно строить планы земли-2.
Аноним 29/10/15 Чтв 19:49:49 #105 №224101 
14461373896130.jpg
14461373896191.jpg
14461373896202.jpg
14461373896213.jpg
>>224096
>что база на полюсе неудобна
Тебе аллели какие-то не позволяют посты читать целиком? Почему ты доебался до шапок, я тебе написал, что воды дохуя прямо в грунте.
Аноним 29/10/15 Чтв 19:52:09 #106 №224102 
>>224098
>Жить под куполом можно и на спутниках гагантов
На спутниках планет - гигантов ты будешь жить не под куполом, а зарывшись в грунт основательно, ибо радиационные пояса ояебу. Давай все-таки ты будешь про аллели, у тебя хорошо получается с ними, а планетология все-таки не твой конек.
Аноним 29/10/15 Чтв 19:58:28 #107 №224103 
>>224101
Потому что ее надо добывать из этого грунта.
>>224102
Каллисто, Ганимед, Титан? Или Европа, Ио и все?
А вообще ты утомил, ты сидишь в спейсаче и мечтаешь никуда не летать?
Аноним 29/10/15 Чтв 20:01:53 #108 №224104 
14461381132950.jpg
Аллели - новый мем спейсача?
Аноним 29/10/15 Чтв 20:03:19 #109 №224105 
14461381995270.jpg
>>224103
>Потому что ее надо добывать из этого грунта.
А воду вообще придется где угодно откуда-то добывать. Она только на Земле собирается сама собой. И это вообще не проблема. Более того, марсианскую воду можно собирать прямо из атмосферы.
>>224104
Нет, это когда биолог приходит в незнакомую ему тему и начинает вещать что-то с умным видом знатока. То есть в биологии он туда-сюда, а астрономии ни сном, ни духом. Получается конфуз.
sageАноним 29/10/15 Чтв 20:03:51 #110 №224106 
>>224084
>1. Объясни это комете.
И что, много было катастрофических (с выпилом всех человеков) падений комет за последние 50 тысяч лет?
Аноним 29/10/15 Чтв 20:05:02 #111 №224107 DELETED
>>224105
>Тред тупых вопросов
>писькомерство кто лучше колонизирует солнечную систему
Да-да, так и есть.
sageАноним 29/10/15 Чтв 20:05:14 #112 №224108 DELETED
>>224088
>найти планету - где можно будет без купола жить
Опять всю ночь фантастику читал?
Аноним 29/10/15 Чтв 20:05:22 #113 №224109 DELETED
>>224106
Спроси у динозавров.
sageАноним 29/10/15 Чтв 20:10:24 #114 №224111 DELETED
>>224109
>50 тысяч лет
>Спроси у динозавров
Реквестирую пикчу про "каменный топор пробивает череп динозавра нахуй".
Аноним 29/10/15 Чтв 20:10:35 #115 №224112 
>>224107
Так вопросов то и не было. Было голословное утверждение с умным видом, что на Марсе мало воды. А вообще платиновый колонизаторосрач зачем устраивать, в треде тупых вопросов.
>>224106
Ну если так прикинуть, то в любой момент жди беды. http://www.passc.net/EarthImpactDatabase/
Аноним 29/10/15 Чтв 20:14:27 #116 №224113 DELETED
>>224111
>Сегодня не упала комета
>За последние 50 тысяч лет не упала комента
>Следовательно комета не может упасть никогда
>>224112
Мир, Союз-Аполлон, жвачка? Просто разочаровался в марсе в плане колонизации, давления нет, солнца нет, нахуй он нужен - не поняттно.
Аноним 29/10/15 Чтв 20:15:55 #117 №224114 DELETED
>>224113
Скажу, что проблема решаемая, соглашусь, что расселяться надо. Человеки вечно куда-то пруться что-то заселяют, от пустынь, до крайнего севера. ДУмаю это где-то в аллелях причина, лол.
Так что я за любые движухи, хоть на Марс, хоть на Альдебаран.
Аноним 29/10/15 Чтв 20:16:57 #118 №224115 DELETED
>>224114
>Прутся
самопочин
sageАноним 29/10/15 Чтв 20:18:55 #119 №224116 DELETED
>>224112
>Ну если так прикинуть, то в любой момент жди беды
Ну если так, то хули все последние века - это не сплошной выживач? Хули сверхглубокие бункеры не копаем, почему законодательно не ограничиваем размеры городов, почему тушенку и воду под землей не складируем?

Да, любой момент может быть пиздец. В любой момент и воздух может собраться в одной половине комнаты, и ты задохнешься. Только вот никто не ходит целый день в маске и с баллоном.
Аноним 29/10/15 Чтв 20:21:27 #120 №224118 DELETED
>>224109
>Спроси у динозавров.
Можно я у млекопитающих спрошу?
Аноним 29/10/15 Чтв 20:24:17 #121 №224119 DELETED
>>224114
Да, я за расселение, но максимально автономное. Чтобы случись что на Земле - не вымерли сразу все колонии. Ну и конечно хотел бы слетать к звездам, в один конец.
>>224116
Ну если так прикинуть мы с точки зрения выживания ведем себя крайне нерационально. Мы не пытаемся максимизировать шансы выжить, лучше новый айфон.
>>224118
Массовое пермское вымирание. 96 % водных видов и 70 % наземных видов вымерли всего за 60 тысяч лет, возможно из-за падения метеорита. Для вымирания людей или отката технологий можно гораздо менее масштабное событие. Мы можем сами спровоцировать климатические изменения, которые вызовут катастрофу.
Аноним 29/10/15 Чтв 20:26:03 #122 №224120 
>>224116
>почему тушенку и воду под землей не складируем?
Ты не поверишь...
>это не сплошной выживач
Ты путаешь прогресс с выживачем.

>В любой момент и воздух может собраться в одной половине комнаты
Статистически ни в какое сравнение с падающими каменюками на голову, коих было очень не мало. Прямо даже скажу, что много одних кратеров диаметром больше 50 км, полтора десятка штук. Это при том, что небольшой кусок говна в 17 м чуть не убил весь Челябинск. Хорошо высоко взорвался.
Аноним 29/10/15 Чтв 20:29:39 #123 №224121 DELETED
>>224119
Причина Пермского вымирания, по последним данным произошла, вероятнее всего из-за как раз Земных причин, а именно извержений Сибирских траппов, хотя вопрос открытый, конечно. Но от этого не легче. Колыбель человечества нам и без астероидов подгадить может так, что мы все охуеем.
Аноним 29/10/15 Чтв 20:33:05 #124 №224122 DELETED
>>224121
Вулканы в тему кометы ну никак не впихивались, а пример показательный. Тем более, что это тоже одна из версий Практически стерилизовало планету, в плане разнообразия видов. А человеку с его нынешней зависимостью от ресурсов, условий и логистики нужно гораздо меньше.
Аноним 29/10/15 Чтв 20:46:21 #125 №224124 DELETED
>>224114
за любой кипишь, кроме голодовки - ок поддерживаю

>>224119 >>224084
я так понимаю биолох, брателло, ну смотри лучше и интересней делать экосистему у хабитата, ты же понимаешь, если чтото пойдет по пизде можно стерилизовать и накатить по новой, с планетами такой фокус мы пока провернуть не можем. давай к нам в сф Очередной тред космических войн перетрем за луну, за хобитаты, за экосистемы.
Аноним 29/10/15 Чтв 20:50:00 #126 №224126 
14461410002740.jpg
>>224121
>>224122
А ведь эпичных вымираний было пять за последних полмиллиарда лет. Мы все под колпаком блджад!
Вполне понятно, почему многим в спейсаче очень чешется куда-нибудь уколонизироваться, даже несмотря на то, что это при их жизни не произойдёт если произойдёт вообще. Пока людишки есть только тут - мы играем в Жизнь без сохранений, ёпта.
Аноним 29/10/15 Чтв 20:58:18 #127 №224129 
14461414989830.jpg
>>224126
Я бы просто свалил бы, без всяких причин и страхов получить космическим камнем в темя. Просто есть часть людей с шилом в жопе.
Нет, в Марс One я не пытался записаться, это изначально афера и идиотизм

>мы играем в Жизнь без сохранений
Веская и обоснованная причина, да.
sageАноним 29/10/15 Чтв 21:05:20 #128 №224132 DELETED
>>224126
>Пока людишки есть только тут - мы играем в Жизнь без сохранений, ёпта.
Твой вид играет в нее всего 1/100000 от всего времени существования жизни на земляшке, а ты уже изнылся.
Аноним 29/10/15 Чтв 21:08:44 #129 №224134 DELETED
>>224132
У него это заложено в генах. Выживание и распространения вида. Все правильно.
Аноним 29/10/15 Чтв 21:15:21 #130 №224137 DELETED
>>224134
>У него это заложено в генах
Скорее, это у него от воспитания. Ерохин не стремится на Жопу Коня, а Сычев, начитавшись в детстве фантастов 60-х, хочет чтобы кто-нибудь (только не он сам, конечно) сел на чугуниевую ракету, и улетел фотки доставлять, для Сычева.

Пардон за /b/-сленг.
Аноним 29/10/15 Чтв 21:16:38 #131 №224138 DELETED
>>224134
>>224137
А кто вам сказал, что я хочу куда-то сьёбывать? Мне и тут норм.
Аноним 29/10/15 Чтв 21:17:02 #132 №224139 DELETED
>>224124
Открыл я тред этот и закрыл, там какой-то пиздец. Про экосистему - интереснее как раз планетарную проектировать, вот реальность терраформинга и инжиниринга экосистем пока как раз уровня /sf.
>>224129
Тоже шило, хотел бы слетать куда-нибудь. Про марс уно думал, но тоже сомнения одолели слишком быстро.
>>224132
Мы можем летать в космос всего 1/100000 от времени существования вида, но это не значит, что надо ждать еще миллион-другой лет, прежде чем начать расселяться.
ВИД СМОГ В КОСМОС
@
ВЫМЕР ОТ МЕТЕОРИТА В ОЖИДАНИИ ВЫХОДА АЙФОНА 101
Аноним 29/10/15 Чтв 21:31:06 #133 №224143 DELETED
>>224137
>Ерохин не стремится на Жопу Коня
А теперь ты всех путешественников и первооткрывателей записал в Сычевы. Ну молодец, чо. Безусловно они все мамкины корзинки, не могущие жить без мамашиной юбки.
Аноним 29/10/15 Чтв 21:36:01 #134 №224144 DELETED
>>224143
А теперь найди мне первооткрывателя на двачах.
Аноним 29/10/15 Чтв 21:41:28 #135 №224146 DELETED
>>224144
У тебя аргумент - инвалид. Первоткрывателя чего? Эпоха великих географических открытий завершилась незадолго до двача, а к космических первооткрывателей нам еще джва года ждать. Найди мне способ поселиться на марсе и я с удовольствием пруфану тебе оттуда.
Аноним 29/10/15 Чтв 21:50:54 #136 №224147 DELETED
>>224146
>Найди мне способ
Вот поэтому ты не первооткрыватель, лел.
А вообще странный диалог у вас, один указал, что другой назвал всех первооткрывателей сычёвыми, другой попросил у него найти первооткрывателя на двоче.
Аноним 29/10/15 Чтв 21:54:12 #137 №224150 DELETED
>>224147
Странный диалог у них, я о том, что до Колумба такие люди тоже были, но без кораблей ты только холодильник можешь попервооткрывать, в перерывах между двачами. На своем бугурте я до марса при всем желании не долечу. Да и не исследователи строили корабли, лел, а корабельщики.
Аноним 29/10/15 Чтв 21:57:32 #138 №224152 DELETED
>>224144
При чем тут двощи, это у тебя аргумент инвалид.
Ты утверждаешь, что Ерохины не стремятся на Жопу Коня, а они как раз стремятся, в том числе современные космонавты. Здоровые и умные, которые прошли отбор среди сотен других кандидатов. Они что, тоже все Сычевы?
Аноним 29/10/15 Чтв 22:00:17 #139 №224154 DELETED
>>224150
А разве свершённые научные открытия не сделают человека "первооткрывателем" в какой-либо области?
Аноним 29/10/15 Чтв 22:02:38 #140 №224157 DELETED
>>224152
И Сычевы стремятся, надо сделать переезд доступным и можно будет оценить миграцию. Я бы свалил. Да, в один конец, да трудно, много работать, условия хреновые.. Но это же охуенно.
>>224154
Тут совсем о другом "путешественников и первооткрывателей", а не "ученых и первооткрывателей", читай тред глазами.
Аноним 29/10/15 Чтв 22:03:05 #141 №224159 DELETED
>>224154
Научные открытия, в основном делаются в лабораториях или еще чаще за кипой бумаг. Это немного другое. Тут можно быть только умным и быть абсолютно не приспособленным для жизни не только на другой планете, а даже просто вне дома-работы, без присмотра жоны или мамки.
Аноним 29/10/15 Чтв 22:04:40 #142 №224161 DELETED
>>224157
То есть первооткрывателем может быть только путешественник?
Аноним 29/10/15 Чтв 22:05:19 #143 №224162 
14461455194490.jpg
>>224139
>ВИD СМОГ В КОСМОС
Аноним 29/10/15 Чтв 22:05:20 #144 №224163 DELETED
>>224161
Блядь, а "коса" может быть только на макушке?
Аноним 29/10/15 Чтв 22:05:38 #145 №224164 
>>224162
Проиграл.
Аноним 29/10/15 Чтв 22:07:23 #146 №224165 DELETED
>>224163
Не только.
Аноним 29/10/15 Чтв 22:07:41 #147 №224166 DELETED
>>224150
Хорошо, что лично ты сделал, чтобы приблизиться к первооткрывателям? Вот Лин ракету пытается пилить, Зеленыйкот - спутник у них нихуя хорошего не выйдет, но они по крайней мере пытаются, куча народу пускают ракеты в стратосферу, пилят кубсаты в универах, устраиваются работать в космическую отрасль, пытаются попасть в космо/астронавты. А что сделал ты, кроме нытья в интернетах о том, что родился слишком рано, чтобы топтать тропинки далеких планет?
Вы заебали уже ныть о том, что кто-то должен доставить вас на Марс. Либо отрывайте жопу от стула, либо заткнитесь.
Аноним 29/10/15 Чтв 22:07:53 #148 №224167 
14461456731760.png
>>224146
>космических первооткрывателей нам еще джва года ждать
Вы огорчаете Плутон.
Аноним 29/10/15 Чтв 22:12:13 #149 №224169 DELETED
>>224166
Дядя Петя, ты дурак? Что ты лично сделал для ДВС, чтобы ездить на машине? Какой твой вклад в создание турбореактивного двигателя, чтобы летать на самолете? Ты хоть одну новую экзопланету открыл, чтобы в спейсач залезть?
Что за хуйню ты несешь? Я готов свои профессиональные навыки применять на марсе, жить там и умереть, но если ты для этого мне предлагаешь разработать, построить, оплатить и совершить перелет самому... Сирзли?
Аноним 29/10/15 Чтв 22:18:03 #150 №224171 DELETED
>>224169
Двачую этого колонизатора.
Аноним 29/10/15 Чтв 22:18:52 #151 №224172 DELETED
>>224169
Поправка: ты готов переехать на Марс если тебе там построят дом и купят билет. Ты нихуя не первооткрыватель, потому что даже тот же Колумб в свое время знатно побегал по королевским дворам, организовывая свою экспедицию, а не дожидался, пока ему все приготовят на блюдечке с голубой каемочкой.
И таких диванных первооткрывателей полон интернет.
Аноним 29/10/15 Чтв 22:19:12 #152 №224173 DELETED
>>224169
какие у тебя проф навыки?
Аноним 29/10/15 Чтв 22:21:38 #153 №224174 DELETED
>>224172
Я готов оплатить часть перелета, пройти тесты, работать на благо в мерзких условиях и жрать питательную пасту с витаминами. А ты что предлагаешь? Построить ракету? Колумб не строил корабль. И его матросы не строили.
>>224173
Асу, Айти.
Аноним 29/10/15 Чтв 22:26:26 #154 №224177 DELETED
>>224173
Ты сейчас переходишь на личности, какая разница, какие у него есть навыки, ты будешь сейчас решать какие навыки нужны, а какие нет? Ты руководитель экспедиции? У тебя есть навыки, ты компетентен решать, кто там пригодится, а кто нет?
Аноним 29/10/15 Чтв 22:26:44 #155 №224178 DELETED
>>224174
> А ты что предлагаешь? Построить ракету? Колумб не строил корабль. И его матросы не строили.
Ну оплати тогда. Организуй. Хоть что-нибудь сделай кроме пиздежа в интернете.
> Асу, Айти.
И нахуя ты нужен на Марсе? Лучше и правда сиди на Земле.
Аноним 29/10/15 Чтв 22:29:11 #156 №224181 
>>224178
Вперед, твоя специальность и чем она нужнее на марсе, чем АСУ. inb4: говном кидаться на двачах - т.к. это не специальность
Аноним 29/10/15 Чтв 22:30:04 #157 №224182 
>>224178
>И нахуя ты нужен на Марсе?
Кто ты такой, что бы решать, нужен он или нет, повторю вопрос? У тебя есть основания решать такие вещи? Ты планетолог? Руководитель экспедиции?
Аноним 29/10/15 Чтв 22:33:47 #158 №224185 
>>224181
Я, если что, никогда не утверждал, что кто-то должен доставить меня на Марс или что я вообще хочу туда.
Специальность, если что - программист. Ничем не нужнее.
>>224182
Ну обоснуй мне, зачем вообще может понадобиться айтишник (даже не АСУ) кроме сисадмина подай-принеси на Марсе/Луне в ближайшие полвека.
Аноним 29/10/15 Чтв 22:37:08 #159 №224186 DELETED
>>224185
Ну и все собственно, кураж исчез, сам слился. Зачем нужны айтишники и асушники там, где будет куча автоматики и обслуживающи роботов... Ну хуууй знает... Может консервы открывать? Только веб-макаки не нужны пожалуй.
Аноним 29/10/15 Чтв 22:37:44 #160 №224187 DELETED
>>224185
>Ну обоснуй мне
Дядя Петя, а ты и правда дурак. Я тебе ничего обосновывать не собираюсь и вообще просил тебя воздержаться от подобных разглагольствований, но почему-то в трех постах ты это не заметил. То, чем ты занимаешься - это переход на личности и демагогия. Ни ты, ни я не компетентны решать, кто нужен на Марсе больше. Особенно, не имея проработанной детально программы.
Так что хватит, остановись, тебя понесло не туда.
Аноним 29/10/15 Чтв 22:47:35 #161 №224190 DELETED
>>224186
> Зачем нужны айтишники и асушники там, где будет куча автоматики и обслуживающи роботов...
И зачем же? Что сможет сделать айтишник, сидящий на Марсе, но не сможет айтишник, сидящий на Земле?
>>224187
> Особенно, не имея проработанной детально программы.
Самая проработанная программа - несколько флаговтыков и домой, до чего-то большего, вполне вероятно, никто из здесь присутствующих не доживет.
Начинал разговор про применимость специальности на Марсе не я (и я зря, на самом деле полез вскрывать эту тему), но я правда не представляю, зачем на Марсе сколь-нибудь развитое IT. Где оно на МКС, где тоже дофига электроники? Проститутку на Марсе я, например, могу представить, хоть это и отдает дешевой фантастикой, а айтишника - нет.
Аноним 29/10/15 Чтв 22:52:22 #162 №224191 DELETED
>>224190
>Не могу представить
>Не знаю
>Буду продолжать доказывать, что просто не нужны.
Аноним 29/10/15 Чтв 22:54:01 #163 №224192 DELETED
>>224191
>Не могу представить
>Не знаю
>Буду доказывать, что может и нужны
Ты же просто засовываешь голову в песок и отказываешься немного подумать.
Аноним 29/10/15 Чтв 22:56:16 #164 №224193 DELETED
>>224190
Ладно, подскажу, но последний раз. Айтишник на земле, и уж тем более асушник на земле должен будет ждать от 42 минут при минимальном отдалении и 202 минут при максимальном чтобы получить ответ, мало того, вся телеметрия должна будет передаваться по радиосвязи, нужна система спутников чтобы эту связь не терять и ни о каком риалтайме речи не идет. Этого аргумента достаточно?
>>224192
>Сам дурак
Ну-ну, аргументы давай.
Аноним 29/10/15 Чтв 22:56:54 #165 №224194 DELETED
>>224193
Блядь, разметка жи 4х2 минут и 20х2 минут.
фикс
Аноним 29/10/15 Чтв 23:00:26 #166 №224195 DELETED
>>224139
это спецовая нычка всего двача, клуб джентельменов обсуждающих и планирующих наше космическое существование пара неадекватных засланцев конечно ворвалось и насрало там, но Per aspera ad astra

>Про экосистему - интереснее как раз планетарную проектировать
У меня для тебя одно слово - Австралия.

Мы климат то посчитать на этой планете не можем, погоду на 2 дня. Количество проблем возникающих на пути оформления нескольких тысяч квадратных километров биосферы хабитата, удовлетворят самого взыкательного садомита. Особенно если добавить, что не смотря на контролируемый по нескольким параметрам климат и зонирование - требуется чтобы совместимость с земной биосферой осталась. В первую очередь на микробилогическом уровне. Для эстетов боли, надо сделать так чтобы человек со станции мог прилетель на землю, и при этом чобы тут все не наебнулось. Вот это вызов, пара десятуов сотен тысяч таких систем и уже можно подумывать о планете, вместо того чтобы ее хлореллой засеивать.

Вангую все ставки на Марс как территорию проживания фигня, Вот Венера может быть интересной темой, сделаем там типо парк юрского периода. Будем туда туризм устраивать и пострелушки Но помни внерианский лесник еще круче того лесникаНу такой по типу хищника
Аноним 29/10/15 Чтв 23:05:44 #167 №224196 DELETED
>>224193
> Айтишник на земле, и уж тем более асушник на земле должен будет ждать от 42 минут при минимальном отдалении и 202 минут при максимальном чтобы получить ответ
Уточню: те, кто пользуются готовой системой тоже считаются айтишниками?
Зачем реалтайм разработчику? Я, возможно, не знаю специфики, потому этот вопрос без сарказма.
> мало того, вся телеметрия должна будет передаваться по радиосвязи, нужна система спутников чтобы эту связь не терять
Эта система будет нужна в любом случае, разве что на флаговтыке можно будет обойтись без полностью непрерывной связи.
> аргументы давай.
Достаточен ли аргумент в виде того, что в обозримом будущем (не беря неожиданные прорывы в космических технологиях) доставить человека на Марс и обеспечить его всем необходимым будет куда дороже, чем оставить его на Земле?
Аноним 29/10/15 Чтв 23:11:42 #168 №224198 DELETED
>>224195
Вангую что в хабитатах будут растить модифицированные водоросли и клонировать мясо без коров, после чего вкусовыми добавками делать это все съедобным, а на серьезное планирование экосистем покладут или положат.
>>224196
Системы полетят готовые, будет ремонт, обслуживание и калибровка орудий. Реалтайм нужен будет для реактора скажем, для систем жизнеобеспечения, пожаротушения и прочего. Там где нельзя подождать минимум 40 минут, пока с земли среагируют.
>без полностью непрерывной связи.
И без канала на хуй знает сколько мегабит, ты же понимаешь, что радиосвязь с другой планетой не только медленная, но и "узкая"? При управлении с земли нужен будет канал пошире, чем для просмотра порнухи на плазме.
>будет куда дороже
Не аргумент, сначала научные цели, они всегда дорогие, потом найдут экономическое обоснование и пойдет-поедет. Но я выше писал, что в именно марсе разочарован. В случае той же Каллисто или Ганимеда задержка связи будет чудовищной, канал тонюсеньким и без специалистов на месте уже совсем никак. Впрочем и на марсе никак, просто можно быстрее получать помощь от спецов с земли.
Аноним 29/10/15 Чтв 23:23:35 #169 №224200 DELETED
>>224172
Колумб ок возможно был в том числе и организатором, но первооткрывателями были и другие сифилитики члены коменды судна, которых ессно никто не помнит на бытовом уровне.
И если бы этих граждан небыло, или в какойто момент они плохо старались не сдохнуть, никто бы колумба не вспоминал.

Но проводить прямую аналогию с коломбом, при всем моем уважении, не корректно. Это эквивалентно утверждению, что на данный момент таких инициативных кадров которые гребут в сторону космоса нет. Со всей очевидностью это лодное утверждение, кто там уже двадцать лет в наса бегает и пытается протолкнуть марсиансукую экспедицию.

В наше время эквивалент колумба это тысяча человек которые бегают и продвигают и эквивалент команды это миллион другой за их спинами.
Даже простое благожелательное отношение несколких миллионов человек по всей планете уже имеет значение и результат, в наше время. Этот факт несколько облегчает проблему, но не делает ее простой. Должно быть организованное, возможно интернациональное движение, если есть желание чтобы вещи двигались, поэтому гнать ссаными тапками человека который готов приложить усилия, пусть и не так или там где ты возможно хочешь это сейчас, это глупо.
Организации нужны все, просто все.
Аноним 29/10/15 Чтв 23:27:20 #170 №224201 DELETED
>>224198
> Реалтайм нужен будет для реактора скажем, для систем жизнеобеспечения, пожаротушения и прочего.
Разве не лучше это делать в любом случае делать максимально автоматизированным?
> И без канала на хуй знает сколько мегабит, ты же понимаешь, что радиосвязь с другой планетой не только медленная, но и "узкая"? При управлении с земли нужен будет канал пошире, чем для просмотра порнухи на плазме.
Да, понимаю.
> Не аргумент, сначала научные цели, они всегда дорогие, потом найдут экономическое обоснование и пойдет-поедет.
Это по-моему дело совсем отдаленного будущего и многотысячных (ну ладно, многосотенных хотя бы) колоний. Просто сейчас содержание одного человека на орбите стоит порядка миллиарда долларов в год, и эта цифра для Марса и чего-то подальше будет только больше. При таких расценках поневоле задумаешься, не выгоднее ли будет посадить заниматься той же работой пару сотен человек на Земле.
В общем, похоже, мы с тобой о сильно разных временных промежутках говорим.
Аноним 29/10/15 Чтв 23:32:18 #171 №224202 DELETED
>>224177
>ты будешь сейчас решать какие навыки нужны
уже решил, нужны все, просто меня интересует спектр тех кто хочет, почему бы и не поинтересоваться, за спрос денег не беру.

>Ты руководитель экспедиции?
я мог бы им быть, хотя ты говоришь не о том о чем я, но даже в твоем понимании.
Аноним 29/10/15 Чтв 23:34:45 #172 №224205 DELETED
>>224201
>делать максимально автоматизированным
Кто это будет делать и кто будет следить, чтобы это не ломалось? Ответ: специалист, который это развернет на месте и будет обслуживать, ну и в критической ситуации будет сидиеть и исправлять/рулить/заглушать ручками.
>Это по-моему дело совсем отдаленного будущего
>содержание одного человека на орбите
Пока мы собираем каждую ракету отдельно (чуть ли не проектируем) - да, в космос надо научиться выходить, и не только раз в год по праздникам. Упирается в вывод на орбиту сильно, дальше там и дельты не такие уж большие и содержание можно удешевить буду жрать мучных червей, если придется.
>>224202
Сорганизовались бы при моей жизни...
Аноним 29/10/15 Чтв 23:50:07 #173 №224209 DELETED
>>224202
>я мог бы им быть
Нет, не мог, я тебя не назначаю на руководителя экспедиции. Вот решил что ты не нужен. Это суть твоей демагогии, так что кушай.
Аноним 29/10/15 Чтв 23:53:34 #174 №224210 DELETED
>>224196
>доставить человека на Марс и обеспечить его всем необходимым будет куда дороже, чем оставить его на Земле?
Доставить Колумба\Магеллана\Беллинсгаузена\Амундсена (нужное подчернуть) и их команды дороже, чем было оставить их дома.
Гагарина дешевле было бы не запускать. Армстронга сотоварищи должны были остаться дома. Нахуй они вообще летали. Хуйня какая-то только бабки потратили. Дуругое дело айфон.
Аноним 29/10/15 Чтв 23:59:11 #175 №224212 DELETED
>>224210
Да и айфон твой хуйня. Пошли вскроемся.
Аноним 30/10/15 Птн 00:00:18 #176 №224213 DELETED
>>224210
Если бы их задачи могли бы быть выполнены с той же эффективностью без присутствия людей - их бы там не было. Всех. Но - не могли. вот сейчас задачи Армстронга решают роботы, хоть и похуже, сильно похуже. А вот профит от одного айтишника на Марсе против 500 на Земле совсем неоднозначен.
Аноним 30/10/15 Птн 00:08:59 #177 №224214 DELETED
>>224190
>но я правда не представляю, зачем на Марсе сколь-нибудь развитое IT.
ты зря так, я просто весь в предвкушении как ктото будет дебажить и тестировать с 20 минутным пингом когда какойнить кондер вспухнет и все пойдет по пизде, держите меня семеро я хочу на это смотреть. Интересно апполон 13 бы выжил при прочих равных, кроме пинга? Последний вкд смотрел на мкс?, а теперь с 20 минутным, тото будет весело.

Так конечно зависит от объемов предполагаемой деятельности, на марсе или где.

>>224198
>Вангую что в хабитатах будут растить модифицированные водоросли и клонировать мясо без коров, после чего вкусовыми добавками делать это все съедобным
Нее, иначе народ скажет нахуй надо, будут проблемы с людьми по всем фронтам(ну его нахуй столько проблем не надо), экосистема нужна в первую очередь как рекреационная зона, для подержания общепланетарного стандарта, плюс первое поколение непревычно к таким условиям. Воообщем, есть основание сделать первую станцию кусочком земли, это проще чем разгребать проблемы-последствия. не исключено что потом оно будет кастрировано до предложенного тобой варианта, но это уже другая сказка

>>224205
>Сорганизовались бы при моей жизни...
может быть может быть, не теряй надежду и не стремайся запрыгнуть на поезд.

>>224209
улыбнул ты меня, желаю успехов с таким подходом, но если что, всегда милости просим к моему шалашу, такие нужны мне, без тени сомнения.
Аноним 30/10/15 Птн 00:11:40 #178 №224215 DELETED
>>224213
>вот сейчас задачи Армстронга решают роботы
Задачи Армстронга роботы не могли решить по определению. Просто на основании того, что они роботы. Напоминаю, что к моменту высадки на Луну по ней уже катались роботы. Так-то.

>желаю успехов с таким подходом
Это твой подход, ты вопил же, что АСУ-шник НИНУЖЕН, или не ты? На каком основании? Ни на каком. Просто пук в небо.
>>224212
Иди вскрывайся, я тебе тут не товарищ.
Аноним 30/10/15 Птн 00:15:33 #179 №224216 
>>224214
40 минутным, в одну сторону 20 минут. "Хьюстон, у нас тут система подачи кислорода наебнулась!" "20 минут назад у вас начались неполадки, ждите, сейчас за 40 минут проверим пару комманд... надо обработать реультат, 5 минут.. И еще 40, мы починили, Марс, как слышите? Марс? Марс??"
Аноним 30/10/15 Птн 00:20:51 #180 №224217 
14461536514810.gif
>>224213
>вот сейчас задачи Армстронга решают роботы
Аноним 30/10/15 Птн 00:29:50 #181 №224218 
>>224215
>Это твой подход, ты вопил же, что АСУ-шник НИНУЖЕН, или не ты?

не я, хотел написать чтобы ты протер детектор, но подумал что это будет не совсем корректно по отношению к не так давно пришедшим, в такой важной для человечества дискуссии. АСУ это вообще первый человек, по всем статьям. Марсианская станция или нет, эксплуатация или разработка-постройка.

>>224216
пинг от 8 до 26 минут если говорить о марсе http://blogs.esa.int/mex/2012/08/05/time-delay-between-mars-and-earth/
расстояние от 0.4 до 2.4 а.е. , я взял среднее не минимум и не максимум
Аноним 30/10/15 Птн 00:32:08 #182 №224219 
>>224167
Хороший пример. Пока работают автоматы.
Аноним 30/10/15 Птн 00:33:24 #183 №224220 
>>224219
Задачи миссии были другие, не путай.
Аноним 30/10/15 Птн 00:37:39 #184 №224221 
>>224218
>Время между отправкой запроса и получением ответа (RTT, от англ. Round Trip Time) позволяет определять двусторонние задержки (RTT) по маршруту
Ок, 8 и 52 минуты минимальный и максимальный пинг. Средний 36.
Аноним 30/10/15 Птн 00:40:24 #185 №224222 
>>224220
Я к тому, что пока что работают автоматы. И в будущем будут работать автоматы, более сложные.
Аноним 30/10/15 Птн 00:41:35 #186 №224223 
>>224221
30 средний.
фикс
Аноним 30/10/15 Птн 00:42:19 #187 №224224 
>>224222
Ты нить беседы потерял, к началу вернись, мы тут про расселение обезьян, а не строго изучение космоса.

Аноним 30/10/15 Птн 00:44:06 #188 №224226 
>>224224
А роботы, кстати, гораздо лучше расселятся... ИИ, технологическая сингулярность, скайнет, матрица, гроб, гроб, кладбище, ТЕРМИНАТОР!
Аноним 30/10/15 Птн 00:47:56 #189 №224228 DELETED
>>224198
>Не аргумент, сначала научные цели, они всегда дорогие
Ок, завтра летим на Ганимед, колонию строить. Экономическое обоснование под этот полет можешь предоставить?

Пока что Олимпиады в днищестранах стОят больше ITER (который куда как более близкая цель), а ты "наука всегда вперед". Да она (утрированно) финансируется по остаточному принципу.
Аноним 30/10/15 Птн 00:53:37 #190 №224229 DELETED
>>224228
Какое обоснование? Лунная программа - просто выкинули деньги, изучение марса - просто выкинули деньги. Вложение в науку оно такое, в долгосрочной перспективе сулит что-то термояд, а при жизни инвестора - просто выкинутые деньги. Во славу науки. Поэтому и по остаточному, хотя вложись мы серьезно в космос еще после окончания гонки - сейчас рассуждали бы с луны, стоит ли на Каллисто строить еще базу, или нет.
Аноним 30/10/15 Птн 00:54:22 #191 №224230 DELETED
>>224228
Ну сейчас в спейсаче будет очередной призыв к изменению строя на технократию.
>>224226
Пока роботы никак не расселяются и ИИ пока И не блещет, строго говоря.
Аноним 30/10/15 Птн 00:57:10 #192 №224232 DELETED
>>224230
Я не против, технократия имеет много плюсов. А ИИ пока не создан, чем ему блистать? Контролируемый термояд вон тоже не блещет.
Аноним 30/10/15 Птн 00:58:00 #193 №224233 
Сколько раз в течение года солнце пересекает границы созвездий?
Аноним 30/10/15 Птн 01:02:02 #194 №224235 DELETED
>>224229
Всё опять скатилось к технофашизму, чаду урановых топок и чугунию: айфоны - плохо, холодная война - хорошо. Хотя те же "айфоны" (во всех проявлениях) дали космосу больше, чем вся гонка вооружений.
Аноним 30/10/15 Птн 01:03:26 #195 №224237 DELETED
>>224232
>Я не против, технократия имеет много плюсов
Ну разумеется ты не против. До тех пор, пока свободы не ограничат, и на завод не погонят.
Аноним 30/10/15 Птн 01:04:21 #196 №224239 
>>224233
13
Аноним 30/10/15 Птн 01:05:15 #197 №224240 
>>224233
>Сколько раз в течение года солнце пересекает границы созвездий?
Ноль.
Аноним 30/10/15 Птн 01:05:55 #198 №224243 
>>224239
>>224240
Как так? Почему мнения разошлись?
Аноним 30/10/15 Птн 01:07:24 #199 №224245 
>>224237
В чем отличие от того, что сегодня? А ну да, на завод не гонят, рабство надо самому на собеседовании добиваться.
>>224235
На айфонии летаем, вестимо. Что космосу дали мирные технологии?
>>224243
Потому что созвездия - выдуманные группы звезд и их границы Солнце не пересекает, а вот видимый на небе путь Солнца за год пересекает 13 созвездий. Как-то так.
Аноним 30/10/15 Птн 01:08:23 #200 №224246 
>>224245
Спасибо, анончик.
Аноним 30/10/15 Птн 01:08:25 #201 №224247 DELETED
>>224235
>Хотя те же "айфоны" (во всех проявлениях)
В каких "Всех" уточни. Не понимаю, что ты еще хочешь в айфоны записать, кроме самого айфона.
Аноним 30/10/15 Птн 01:09:03 #202 №224248 
>>224243
>Как так? Почему мнения разошлись?
Ну, знаешь, такое иногда бывает - мнения людей расходятся. А может это ты долбоёб, и некорректно сформулировал вопрос.
Аноним 30/10/15 Птн 01:10:12 #203 №224249 
>>224248
Видимо корректно, раз умный анончик уже ответил. А глупый анончик только и может, что обвинять во всем вопросы.
Аноним 30/10/15 Птн 01:10:24 #204 №224250 DELETED
>>224247
>Не понимаю
Это бывает.
Аноним 30/10/15 Птн 01:10:44 #205 №224251 DELETED
>>224247
Ну он хочет сказать, что мирный Фон Браун, изобрел мирные ракеты, потом мирный атом на мирных МБР открыл дорогу в космос и началось мирное соревнование за господство/превосходство (мирное) в космосе, где мирные коспьютеры на мирных спутниках-шпиёнах... И так далее.
Аноним 30/10/15 Птн 01:11:51 #206 №224252 
>>224249
>>224248
Вы оба в треде тупых вопросов, хули на вентиллятор продолжаете набрасывать? Какой тред, такие и вопросы/ответы.
Аноним 30/10/15 Птн 01:12:10 #207 №224253 
>>224249
А ты смешной. Сам решил, кто умный, а кто глупый. По одному ответу хуй знает кого на анонимой борде.
Аноним 30/10/15 Птн 01:14:24 #208 №224256 DELETED
>>224251
И сейчас ты пишешь с прототипа сверхсекретного военного компа?
Аноним 30/10/15 Птн 01:16:52 #209 №224258 DELETED
>>224256
Т.е. ты подменяешь создание компов (где военные цели очень даже превалировали), и последующую оптимизацию, и "типа если б не айфон компов бы не было"? Оптимизация мирными технологиями - это нормально, но не мирные технологии дали космосу всё это.
Аноним 30/10/15 Птн 01:17:19 #210 №224259 
>>224224
>2015
>расселение обезьян
А, это. Я и забыл, что это тренд тупых вопросов.
Аноним 30/10/15 Птн 01:18:44 #211 №224261 DELETED
>>224256
Ты хоть бы погуглил, зачем эти компы придумали вообще и для чего применялись, прежде чем делать столь категоричные заявления.
Аноним 30/10/15 Птн 01:22:47 #212 №224262 DELETED
>>224256
Ну смотри:
1. Химический реактивный двигатель. Военный.
2. Компьютеры и бортовая электроника для космоса. Военные + гонка.
3. Системы ориентации и слежения, коррекции курса - айфон. Шучу, перекочевали из ракет с боеголовками.
Что еще? РИТЭГ? Космические ядерные реакторы? Что мирное-то изначально там летает?
Аноним 30/10/15 Птн 01:24:50 #213 №224263 DELETED
>>224261
Я погуглил за него:
Начало 1943 года — успешные испытания прошла первая американская вычислительная машина Марк I, предназначенная для выполнения сложных баллистических расчётов американского ВМФ.
Конец 1943 года — заработала британская вычислительная машина специального назначения Colossus. Машина работала над расшифровкой секретных кодов фашистской Германии.
Аноним 30/10/15 Птн 01:25:59 #214 №224266 DELETED
>>224262
Сапфировое стекло? Ах да, я забыл, тоже сначала в боевых ракетах использовали.
Аноним 30/10/15 Птн 01:27:25 #215 №224268 DELETED
>>224266
Яблодрочеры, такие яблодрочеры.
Аноним 30/10/15 Птн 01:30:06 #216 №224269 DELETED
>>224261
Да всё это я прекрасно знаю. Только 2 момента:
1. Даже без военных целей то, что может быть создано, создано будет. Да и помнится, Тьюринг, фон Нейман и другие печатали свои статьи в открытых источниках
2. Бюджет многих "компьютерных" корпораций уже превышает бюджеты стран. Стало ли бы это возможно без потреблядства?

И не надо приводить аргументы типа "а вот если бы..." (про сослагательное наклонение и историю, надеюсь, помнишь).
Аноним 30/10/15 Птн 01:33:07 #217 №224271 DELETED
>>224269
Если бы как раз у тебя, если ты не заметил.
>то, что может быть создано, создано будет
Это как раз и есть "если".
Могло бы, может быть, если, но создалось в военных целях, так что про сослагательное наклонение и историю, надеюсь, помнишь
Аноним 30/10/15 Птн 01:34:43 #218 №224274 DELETED
>>224269
Предпосылки к созданию атомной бомбы Эйнштейн сделал тоже вполне мирно, но нельзя говорить, что мирная наука дала нам проект Манхэттен первую ядрену бомбу. И бюджет корпораций не дал нам компьютеры, он дал нам пекарни в Ведьмака погонять, компьютеры нам дала война. И точка, тут нет сомнений и разночтений. Если бы войны не было - это ты начинаешь. Война была, компьютер создан.
Аноним 30/10/15 Птн 01:39:23 #219 №224278 DELETED
>>224271
Нет, у тебя проблемы с логикой:
- если бы посильнее напряглись, построили бы за 50 лет
- если это создаваемо, то будет построено (хоть в виде прототипа) в отдаленном будущем

Чувствуешь нетождественность этих высказываний?
sageАноним 30/10/15 Птн 01:40:28 #220 №224280 DELETED
>>224274
>не дал нам компьютеры, он дал нам пекарни в Ведьмака погонять
Санитаров.
Аноним 30/10/15 Птн 01:43:09 #221 №224281 DELETED
>>224278
Твои слова:
>И не надо приводить аргументы типа "а вот если бы

Следующий пост твой же:
>Построили бы
>если бы

Чувствую, что ты начал вилять задом, вот что. Сначала нес про сослагательное наклонение в истории, потом тебя тыкнули носом в него же, и тут вдруг РАЗНИЦА нарисовалась. Нет уж.
Аноним 30/10/15 Птн 01:44:21 #222 №224282 DELETED
>>224280
Что не так сказал? Пе(рсональный)ка(мплюхтер) и есть плод корпорации, все основы заложены были до. Ведьмака даже не ставил, если ты об этом, я больше по песочкам.
Аноним 30/10/15 Птн 01:47:29 #223 №224284 DELETED
>>224278
Аноним 30/10/15 Птн 01:53:22 #224 №224285 DELETED
Вечерний и то был интереснее ночного. Всем звездной ночи в этом тренде.
технократ-марсианин
Аноним 30/10/15 Птн 01:55:43 #225 №224286 DELETED
>>224285
Ну извини, мы тут не нанимались тебя развлекать, что поделать.
Аноним 30/10/15 Птн 02:03:17 #226 №224287 DELETED
>>224284
>wm-joke spotted
Все укрываемся в складках местности!
sageАноним 30/10/15 Птн 02:08:30 #227 №224289 DELETED
>>224281

а ты правда туповат
Аноним 30/10/15 Птн 02:22:59 #228 №224291 DELETED
>>224289
Дятел, перестань, ты сам себе в штаны нагадил.
Аноним 30/10/15 Птн 02:23:22 #229 №224292 
Предположим я сяду на корабль и буду ускоряться со скоростью немногим больше 1g (1,3g, например) постоянно, предположим медицина позволит мне пребывать в полете 100 лет, например с периодами криосна, не важно. Я ведь смогу, скажем нарезая круги перпендикулярно плоскости эклиптики млечного пути сначала посмотреть как люди расселятся, потом увидеть взлет и падение человеческой цивилизации, потом как гаснут звезды и в итоге увидеть смерть вселенной?
Аноним 30/10/15 Птн 02:25:44 #230 №224293 
>>224292
Время замедлится, но не остановится, так как достичь с все равно нельзя, и ста лет по твоим часам не хватит.
Аноним 30/10/15 Птн 02:28:53 #231 №224296 
>>224293
548071677170000 лет теоретически, где я ошибся?
Аноним 30/10/15 Птн 02:32:30 #232 №224297 DELETED
>>224281
Ты не ответил на вопрос. Тождественны ли эти два высказывания?
Аноним 30/10/15 Птн 02:32:37 #233 №224298 
>>224292
Нарезать круги не получится, потому что плоскость траектории должна проходить через центр масс галактики.
Аноним 30/10/15 Птн 02:38:22 #234 №224299 
>>224298
В смысле? Почему я не могу летать по "полярной" орбите вокруг центра галактики? Так чтобы можно было достаточно равномерно наблюдать за людьми, на начальном этапе (потом скорее всего вымрут и наблюдать можно будет, как гаснут звезды, или там столкновение с туманностью Андромеды).
Аноним 30/10/15 Птн 02:42:02 #235 №224301 
Еще вопрос, скажем я буду накачивать водород в пустое пространство, но понемногу, не допуская резкого сжатия и нагрева облака, можно ли накачав водорода ~1 массы солнца добиться того, чтобы оно не зажглось, т.е. получить газовый гигант с массой позволяющей в термояд, но не зажженый? И можно ли потом его поджечь, лазером или термоядерной бомбой?
Аноним 30/10/15 Птн 02:50:51 #236 №224303 
Луна вращается вокруг Солнца?
Аноним 30/10/15 Птн 02:56:48 #237 №224304 
>>224303
Луна вращается вокруг Земли, которая вращается вокруг Солнца. Так что да.
Аноним 30/10/15 Птн 02:58:09 #238 №224305 
>>224221
да максимальный я существенно промахнулся pong
Аноним 30/10/15 Птн 02:59:36 #239 №224306 DELETED
>>224297
Ты занимаешься демагогией, в демагогии я не участвую. Это пустое.
Ты вступаешь сам с собой в противоречия и задаешь при этом нелепые вопросы как бы вынуждая оправдываться за твои же косяки.

Твои слова:
>И не надо приводить аргументы типа "а вот если бы

Следующий пост твой же:
>Построили бы
>если бы

Все сказано этим. Если тебе норм это, то удачи.

Аноним 30/10/15 Птн 03:13:02 #240 №224308 DELETED
>>224228
в чем проблема, аноны собираются запиливают производства и тратят прибыл не на девок и олимпиады, а на свою страсть и зарплаты.

>>224269
>Да и помнится, Тьюринг, фон Нейман и другие печатали свои статьи в открытых источниках
Криптографы у нас подписку дают ? Так чето вспомнилось, ктото мне втирал.

>>224301
интересный вопрос, если будешь подвозить его грузовиками и высыпать на поверхность и давать всему этому остыть, то может и не получиться или время закончится
Аноним 30/10/15 Птн 03:14:36 #241 №224310 
>>224303
да, влияние солнца на луну больше чем влияние земли на луну, земля это возмущающий фактор во вращении луны вокруг солнца.
Аноним 30/10/15 Птн 03:15:04 #242 №224311 
>>224301
Зависит от времени, на которое ты рассчитываешь. Ну а так да - газопылевые облака существуют. Они эволюционируют, со временем, но такое возможно.
Аноним 30/10/15 Птн 03:19:01 #243 №224312 
>>224296
А какую скорость себе поставил?
Кстати, на счет увидеть: будет сложно на такой скорости, очень сузится поле зрение и все посинеет навстречу.
Аноним 30/10/15 Птн 03:19:33 #244 №224313 DELETED
>>224306
>Следующий пост
Ты выдергиваешь цитату из первого (не обязательно верного) утверждения. Ты - уходишь от ответа. Так тождественны ли эти (пусть, допустим, и оба неверные) высказывания?
Аноним 30/10/15 Птн 03:21:59 #245 №224314 DELETED
>>224308
>Криптографы у нас подписку дают ?
Понятия не имею. Но фоннеймановское определение алгоритма - в каждом втором ВУЗе изучают.
Аноним 30/10/15 Птн 03:30:28 #246 №224316 
>>224310
>да, влияние солнца на луну больше чем влияние земли на луну, земля это возмущающий фактор во вращении луны вокруг солнца.
Define тогда "вращение".
Аноним 30/10/15 Птн 03:38:33 #247 №224319 
14461655133660.jpg
14461655133701.jpg
>>224301
>я буду накачивать водород в пустое пространство
Зависит от объема пустого пространства. Протяженное облако может быть и в сотни солнечных масс, как тут уже говорили.
Однако, если будет ограниченный объем, то коллапс неизбежен. Не бывает абсолютно однородного распределения массы, где-то обязательно будет перекос и дальше будет рождение звезды.


>>224303
Для наглядности тебе две картинки нашел.
Аноним 30/10/15 Птн 03:41:17 #248 №224321 
>>224316
У нас тут не все астрономию изучали, многие путают вращение и обращение.
Аноним 30/10/15 Птн 03:52:16 #249 №224322 
>>224319
На второй движение относительно орбиты Земли значительно менее выражено, чем движение вокруг звезды. Ну а так да - всё вращается вокруг центра масс хуй знает чего. Для удобства выделяют некоторые "точки".
Аноним 30/10/15 Птн 03:56:27 #250 №224323 
>>224299
Тогда другое дело. Упрощаешь галактическое движение до кеплеровского, решаешь уравнения Лебедева в безразмерном виде для малых постоянных возмущений орбиты. Это нетрудно.
Аноним 30/10/15 Птн 05:03:13 #251 №224327 DELETED
В вэ сегодня отрыв по вашей теме. Заходите на огонёк поржать:
https://2ch.hk/v/res/25343.html
https://2ch.hk/v/res/32078.html
Аноним 30/10/15 Птн 05:19:19 #252 №224328 DELETED
>>224327
>восстановить жизнь после того как солнце столкнётся с землёй
Ну не знаю. Я бы спермы полбанки слил, для опытов.
Аноним 30/10/15 Птн 05:25:00 #253 №224329 
>>224316
Сорри оговорился, конечно речь об орбитальном движении луны вокруг солнца
Аноним 30/10/15 Птн 05:45:38 #254 №224330 
>>224329
Сложная кривая, которую приблизительно можно считать окружностью. Тебе для какого-то исследования или статьи нужно точнее?

В целом, относительные расстояния там так великИ, что можно забить на всё (обращение вокруг Земли, прецессию, нутацию, и еще десяток факторов).
Аноним 30/10/15 Птн 05:56:29 #255 №224332 
>>224330
>окружностью
Эллиптической кривой, ёпта. Никак не могу картинки из детских книжек из головы выбить.

Может это и хорошо, что книжки такие читал/смотрел?
Аноним 30/10/15 Птн 06:10:22 #256 №224334 
>>223277 (OP)


Правильно ли я понимаю, что для появления на космическом судне притяжения, эквивалентного земному, оно должно двигаться с ускорением 9,8 м/с.кв?
Аноним 30/10/15 Птн 06:31:17 #257 №224336 
14461758774460.jpg
>>224334
>Правильно ли я понимаю, что для появления на космическом судне притяжения, эквивалентного земному, оно должно двигаться с ускорением 9,8 м/с.кв?
Очень тупая статья (засранная, к тому же, альтернативщиками), но всё же:
https://ru.wikipedia.org/wiki/Принцип_эквивалентности_сил_гравитации_и_инерции
Аноним 30/10/15 Птн 07:32:08 #258 №224344 DELETED
>>224157
>да трудно, много работать, условия хреновые.. Но это же охуенно.
Так и представил, как мамкин корзиноид, после двенадцатичасовой смены по обслуживанию какой-нибудь термальной турбины плачется на локальном дваче с парочкой таких же долбоебов в "дно-треде" о том, что романтика космоса уплыла, исследуют поверхность альфачи Ерохины, а ему остается только вилкой чистить турбину за норму пайки. А мог бы на земляшке остаться у мамки под юбкой и двачевать большую активную капчу.
Аноним 30/10/15 Птн 08:07:30 #259 №224350 
>>224312
Какую скорость? Ускорение 1,3g, постоянное, скорость будет пытаться подобраться к световой.
Аноним 30/10/15 Птн 08:11:35 #260 №224351 
>>224312
Ну сузится, доплер и синее впереди и красное сзади (а потом сзади вообще мало что будет догонять) понятно, под увидеть подразумеваю обработку на компьютере.
Аноним 30/10/15 Птн 08:14:23 #261 №224352 
>>224334
Можно просто завращать его, например сделав жилые палубы в торе перпендикулярном продольной оси.
Аноним 30/10/15 Птн 08:28:06 #262 №224354 DELETED
>>224344
>Я в стотысячный раз пошутил на двачах на тему корзинок. Мам, ну смешно жи, да?
Аноним 30/10/15 Птн 08:58:38 #263 №224356 DELETED
>>224210
Ты заебал уже со своим айфоном, сука! Сразу видно залетного. Разработка новых новых телефонов точно также двигает прогресс. Пусть он и замедлен реалиями рынка, но уж какой есть. Ты вообще смекаешь, что человечество не на твои хотелки работает, а на усредненные хотелки платежеспособных индивидуумов. И если эти хотелки игнорировать, то вся система рухнет нахуй, не то, что в космос никто не полетит.
Аноним 30/10/15 Птн 09:29:47 #264 №224361 DELETED
>>224356
>КО-КО-КО если мне не дать новый айфон - ВСЯ СИСТЕМА РУХНЕТ, мы все умрем!
Что ты несешь, поехавший, прогресс сейчас замедлен реалиями рынка, тут соглашусь, но разработка телефонов и прочей прикладной хуйни двигает фундаментальные науки? Или под прогрессом ты понимаешь то, что у тебя в телефоне теперь не тетрис, а 100 энгри бердсов? Да, это что-то улучшило, но ежегодно мы на это тратим ресурсов больше, чем тратили за столетия еще несколько веков назад, в первую очередь энергоносителей. Человечество работает на хотелки, а хотелки это не космос или термояд, это чипсы, пивас и резиновая баба с вибромотором в жопе. И емли продолжать работать на хотелки - вот тогда система однажды может рухнуть, когда не хватит ресурсов.
Аноним 30/10/15 Птн 09:44:10 #265 №224362 DELETED
>>224361
Понятно, ты идеалист дохуя, разговор исчерпан.
Аноним 30/10/15 Птн 09:44:49 #266 №224363 DELETED
>>224361
Больше чугуния богу чугуния!
Аноним 30/10/15 Птн 09:49:52 #267 №224364 DELETED
>>224362
А у тебя маркетинговый хуй слишком глубоко в жопе, разговор исчерпан. Вон выше уже один обдоказывался, что айфон дал ракетам больше, чем вся гонка и война. До сих пор отстирывается. Не ты?
Аноним 30/10/15 Птн 10:00:26 #268 №224366 DELETED
>>224364
Ну разумеется. Война и пиздошенье палками друг друга по голове создает ракеты. Ты вообще из наркотрипа выходишь?
Аноним 30/10/15 Птн 10:08:43 #269 №224370 DELETED
>>224366
Нет, что ты.
В Индии в конце XVIII века ракетное оружие применялось весьма широко, и, в частности, существовали особые отряды ракетчиков, общая численность которых достигала примерно 5000 человек.

В начале XIX века армия также приняла на вооружение боевые ракеты, производство которых наладил Уильям Конгрив (Ракета Конгрива).

17 августа 1933 года была запущена ракета «ГИРД 9», которую можно считать первой советской зенитной ракетой.

В Германии подобные работы вело Немецкое Общество межпланетных сообщений (VfR). 14 марта 1931 член VfR Йоханнес Винклер осуществил первый в Европе удачный запуск жидкостной ракеты.

В VfR работал Вернер фон Браун, который с декабря 1932 г. начал разработку ракетных двигателей на артиллерийском полигоне германской армии в Куммерсдорфе. Созданный им двигатель был использован на опытной ракете А-2, успешно запущенной с острова Боркум 19 декабря 1934 г. После прихода нацистов к власти в Германии были выделены средства на разработку ракетного оружия, и весной 1936 г. была одобрена программа строительства ракетного центра в Пенемюнде, руководителем которого был назначен Вальтер Дорнбергер, а техническим директором — фон Браун. В нём была разработана баллистическая ракета А-4 с дальностью полёта 320 км. Во время Второй мировой войны 3 октября 1942 г. состоялся первый успешный запуск этой ракеты, а в 1944 г. началось её боевое применение под названием V-2.

В 1957 г. в СССР под руководством Сергея Королёва как средство доставки ядерного оружия была создана первая в мире межконтинентальная баллистическая ракета Р-7, которая в том же году была использована для запуска первого в мире искусственного спутника Земли. Так началось применение ракет для космических полётов.
Аноним 30/10/15 Птн 10:12:33 #270 №224371 DELETED
>>224366
Давай проясним. Я не Хайнлайн, не милитарист, не считаю, что пиздошенье палками - это дохуя прогрессивно, но каким поехавшим надо быть тобой, чтобы утверждать, что ракеты продукт мирных технологий? Или что мирные технологии в условиях рынка дохуя (или хотя бы равно) эффективны вкладу военных технологий в фундаментальную науку?
Аноним 30/10/15 Птн 10:16:38 #271 №224372 DELETED
>>224370
Какую крупномасштабную войну вел СССР в 1957г.? Ответ "холодную" не принимается.
Аноним 30/10/15 Птн 10:19:19 #272 №224373 DELETED
>>224372
Холодную, лол, была бы "горячая" - мы бы сейчас не разговаривали. А свои фантазии, что типа холодная война - это не война - это мир (свобода - это рабство, а незнание - сила) оставь при себе. И Советы и США тогда трудились исключительно над мирными проектами, Мань.
Аноним 30/10/15 Птн 10:19:51 #273 №224374 DELETED
>>224371
Сколько неиспытательных запусков ракет военного назначения сейчас производится в год?
Аноним 30/10/15 Птн 10:20:52 #274 №224375 DELETED
>>224373
Слился.
Аноним 30/10/15 Птн 10:21:58 #275 №224376 DELETED
>>224372
МБР - это же мирная ракета, ты же понимаешь. Межконтинентальный доставщик ромашек.
>>224374
Не виляй жопой, что дало нам ракеты - война, что дало нам компьютер - война, список можно продолжать, а ты начинаешь вилять и переходить к количеству запусков, и прочей демагогии.
>>224375
Тебе в рот слился, не неси хуйни прост.
Аноним 30/10/15 Птн 10:30:57 #276 №224379 DELETED
>>224376
Да пиздец какой-то. 70 лет не воюем всем миром, а гляди-ка - развиваемся.
Аноним 30/10/15 Птн 10:39:05 #277 №224381 DELETED
>>224379
А зачем нужна мировая война, чтобы двигать технологии? Холодной и местных конфликтов (а также всякого сдерживания и ПРО) тебе мало? Или это тоже мирные технологии? Или ты виляешь еще сильнее и теперь "война" понимаешь буквально, а "айфон" иносказательно? Ну и для затравки, давай-ка назови хотя бы 3 мирных космических достижения за эти условные 40 лет с начала разоружения (для мирных целей)? Только без "улучшили и сделали более компактным", принципиально новое что?
Аноним 30/10/15 Птн 10:52:10 #278 №224383 DELETED
>>224381
>3 мирных космических достижения
Наверняка "Пионеры" и "Вояджеры" несут на борту яденые боньбы.
Аноним 30/10/15 Птн 10:55:40 #279 №224384 DELETED
>>224383
Ок, исследование дальнего космоса, но это только один, а не три? Слабовато. Ты сам-то можшь поддержать дискуссию или только вилять жопой вокруг айфона горазд?
Аноним 30/10/15 Птн 11:53:34 #280 №224393 DELETED
>>224384
Cassini, Dawn, New Horizons, SOHO. - все несут по бомьбе.
Аноним 30/10/15 Птн 11:58:07 #281 №224394 DELETED
>>224384
Кстати, это уже просто глупо: их ("Пионеров" и "Вояджеров") - четыре. Ты даже элементарных вещей не знаешь.
Аноним 30/10/15 Птн 12:11:36 #282 №224398 DELETED
>>224394
Ок, а первых бомб было две, поэтому считаем: открытие атомной бомбы 2 (два) раза.
Пионеры + Вояджеры: 4 (четыре) мирных космических достижения. Охуенная логика, умный блин. Опять количество качеством подменяешь?
Аноним 30/10/15 Птн 12:13:13 #283 №224400 DELETED
>>224393
Короче ясно, можешь в виляние и сарказм, а вот в аргументы только на 33,3(3)% процента. НО ВЕДЬ АЙФОН БОЛЬШЕ ДАЛ КОСМАСУ. Что же ты не можешь это больше даже назвать-то?
Аноним 30/10/15 Птн 12:14:52 #284 №224402 
>>224398
Качество количеством т.е.
почин
Аноним 30/10/15 Птн 12:37:24 #285 №224409 
Пиздец вы тут порнуху развели. >>223777 ты в следующий раз отдельный тред создавай, лол.
Аноним 30/10/15 Птн 12:42:30 #286 №224411 
>>224409
Так он правильно создал, тупой вопрос и ответ в 1 строку.
Аноним 30/10/15 Птн 12:50:36 #287 №224414 
>>224409
Не надо создавать, есть спецтред для экзопланет
sageАноним 30/10/15 Птн 12:53:40 #288 №224416 
>>224374
Десятки. Парочка баллистических пусков среди них есть всегда.
Аноним 30/10/15 Птн 12:55:08 #289 №224418 
>>224411
И родил срач на неделю. Хлебом не кормите, дайте только поспорить. Больше бы таких тупых вопросов.
Аноним 30/10/15 Птн 12:57:14 #290 №224421 
>>224418
Его пост никакого срача не родил, экзопланеты в других галактиках не наблюдаются сейчас никак, срач возник вокруг экзопланет рядом, с дистанциями менее 100 с.л.
Аноним 30/10/15 Птн 12:59:40 #291 №224422 
14461991800320.jpg
>>224418
Может нам создать "Нукапошливыйдем-тред", куда перекидывать все вышедшие из под контроля срачи?
Аноним 30/10/15 Птн 13:00:03 #292 №224423 
>>223282
Нейтрино, гравитационные волны.
Аноним 30/10/15 Птн 13:01:33 #293 №224424 
>>224421
>экзопланеты в других галактиках не наблюдаются сейчас никак
http://arxiv.org/abs/0906.1050
Аноним 30/10/15 Птн 13:02:40 #294 №224425 
Спейсачичек, дай ссылок на аудиокниги по космологии.
Послушал КИВ нашего драгоценного Wheelman'a.
Хочется и дальше обернуться в рассказы о космических кораблях на просторах большого театра.
Аноним 30/10/15 Птн 13:08:30 #295 №224426 
>>224422
Модеру нужно тогда создать скрипт, по которому (к примеру) +50 связанных ответов перенаправляются в "Нукапошливыйдем-тред".
Аноним 30/10/15 Птн 13:23:41 #296 №224454 
14462006211210.jpg
>>224426
>Моче
>создавать

>>224424
Спасибо, не знал. Только ближайшая к нам Андромеда, только гиганты 2хЮпитер, но хоть что-то.
Аноним 30/10/15 Птн 13:38:39 #297 №224467 
>>224454
Я идеалист, десу.
Аноним 30/10/15 Птн 17:13:10 #298 №224517 
>>224424
> the possibility to detect extrasolar planets in M31
Аноним 30/10/15 Птн 17:15:04 #299 №224519 
>>224422
Был тред дидосрача например, когда тот всплывал в каждом треде.
Аноним 30/10/15 Птн 17:47:35 #300 №224531 
>>224330
не ты меня с другим аноном попутал это было уточнение, как правильно отметил >>224316 вращение не однозначно определяет о чем идет речь

но если, у тебя вдруг есть под рукой линка на симулятор луна земля солнце я бы посмотрел. Если хочешь упомянуть чтото интересное относительно орбитальной механики системы земля-луна-солнце, то я тоже не откажусь почитать, больше частных производных, Лагранжианов всего этого применительно к сабжу.
Но это все так если под рукой.
Аноним 30/10/15 Птн 21:51:12 #301 №224575 
14462310724410.jpg
Поясните, правда что если в невесомости я замру на месте и не смогу достать до чего-нибудь, то я останусь на месте навечно и так умру?
Аноним 30/10/15 Птн 21:54:53 #302 №224577 
>>224575
Ну можешь бросить что-нибудь. Если сделаешь это правильно - полетишь в противоположную сторону. Поссать, на худой конец.
Аноним 30/10/15 Птн 22:16:31 #303 №224583 
>>224575
В вакууме или в воздухе? В воздухе есть варианты, например плыть как в воде, пусть и получится очень медленно. В вакууме тебе выше написали, надо что-то оттолкнуть/бросить.
Аноним 30/10/15 Птн 22:20:43 #304 №224585 
>>224583
> плыть
А это реально? Ну т.е. вариант напрашивается, да, но у кого-нибудь получалось?
Аноним 30/10/15 Птн 22:30:13 #305 №224588 
>>224585
Не знаю, на моей тарелке есть генератор гравитации. А так должно получиться, ведь разница в сопротивлении при медленном и быстром махе есть, плюс поверхность ладоней, только сила там маленькая, и завращает поди от неосторожного движения. Но если потренироваться - почему бы нет?
Аноним 30/10/15 Птн 22:31:01 #306 №224589 
>>224585
Можешь посмотреть видео с борта МКС, правда чаще всего они там отталкиваются, это эффективнее, воздух очень разряжен, так что плавать не очень умная идея, но да, само по себе возможно, иногда машут руками, что бы дотянуться до чего-нибудь, например.
Аноним 30/10/15 Птн 23:15:14 #307 №224603 
14462361146300.jpg
14462361146341.jpg
Такой вопрос - в союзе/России принципиально не запускали пилотируемые корабли на гептилоблядских ракетах, потому-что яд и всё такое.
В США же в своё время запускали все Gemini на Титанах, заправленных под завязку не менее "приятным" аэрозином. Почему американцы не побоялись пускать людей на таком опасном топливе? Или почему наши так и не решились? Проясни анон, будь добр.
Аноним 30/10/15 Птн 23:16:48 #308 №224604 DELETED
>>224603
Потому что у нас контроль качества на порядок хуже.
Аноним 30/10/15 Птн 23:51:21 #309 №224612 
>>224603
Как это ни странно, но СССР переживали за своих ребятишек куда больше. Ковбои вообще отчаянно рисковали не забывая устраивать реверсы про ко-ко-ко ненадежные Востоки-Восходы. В шатолах безопасность ребятишек порезали до уровня опасного циркового представления: пороховые бустеры нельзя отключить в случае аварии, катапульт и САС(это на самом деле нереально сделать в формате шатола-бурана) нЭт.
А вот СССР на том же Протоне так и разу и не рискнули запускать ребятишек. При том что ТКС - пилотируемый.
>>224604
На парашу проследуй.
Аноним 31/10/15 Суб 00:06:15 #310 №224616 
>>224612
Ойвэй. Королёв просто категорически не хотел (не мог?) пилить пилотируемые ракеты на токсичном топливе, и всячески лоббировал свои взгляды, пользуясь административным ресурсом, что вообще дикость и корень многих зол в совковой космической программе. А проекты-то были.
Ну и инцидентов уже прилично набралось к этому времени, да.
Аноним 31/10/15 Суб 00:17:23 #311 №224619 
>>224616
Королев НЕ ХОТЕЛ гептила потому что понимал опасность при аварии для ребятишек. Это только таким дегенератам как ты, даже сейчас что-то непонятно.
Аноним 31/10/15 Суб 00:57:20 #312 №224628 
>>224619
На самом деле, в герметичной капсуле, отлетевшей на пару километров, как-то похуй, равно как и в бункере на еще большем удалении от старта. А вот хуйни при обслуживании военных ракет хватало, да, но это было уже потом, во времена шахт и быстрого старта.

Кстати, напомни, почему Р-7 была на таком неудобном для МБР топливе? Тоже "ребятишки"?

Американцы летали на конверсионных ракетах, потому и топливо такое. Сатурн 5 уже не юзал его, ибо эффективность так себе.

Гугли, на чём летали разные поколения МБР, и смотри, что было важно в разное время.
Аноним 31/10/15 Суб 01:47:31 #313 №224639 
14462452511420.png
14462452511891.png
14462452512372.png
14462452512853.png
>>224336

Благодарствую!
Аноним 31/10/15 Суб 01:53:08 #314 №224640 
>>224628
Не разговаривай сам с собой. Ты понял о каких ребятишках идет разговор. Н-1 vs упоротый Челомей, если тебе надо намекать. Не только МБР но и зенитные и тактические. Этого говна наелись до отвала. И вообще, сам пиздуй в гугл.
Аноним 31/10/15 Суб 02:25:52 #315 №224648 DELETED
>>224308
>в чем проблема, аноны собираются запиливают производства и тратят прибыл не на девок и олимпиады
Много запили? Много потратили?
Аноним 31/10/15 Суб 02:46:56 #316 №224649 
>>224425
>книги по космологии
>рассказы о космических кораблях
Читать определения терминов.
Аноним 31/10/15 Суб 02:58:29 #317 №224651 DELETED
>>224640

СССР
@
ЗАБОТИЛСЯ
Аноним 31/10/15 Суб 03:13:11 #318 №224653 DELETED
>>224651
То есть башни САС на Союзах лепили для красоты? Или сейчас будет очередной спич про то, какой совок был плохой?
Аноним 31/10/15 Суб 03:55:48 #319 №224662 DELETED
>>224653
>Или сейчас будет очередной спич про то, какой совок был плохой?
Очередной - это какой по номеру? И номер поста, будь добр, приведи.
Аноним 31/10/15 Суб 04:17:01 #320 №224665 
Патцаны, поясните за Интерстеллар.

В вики указано следующее:
Неожиданно по радиосвязи с Купером связывается ТАРС, который сообщает, что они попали в пятимерное пространство. Купер понимает, что таинственная сверхцивилизация — это человечество из будущего, преодолевшее границы пространства-времени и теперь помогающее самому себе в прошлом.

Значит человечество из будущего, как-то пережило эту угрозу и вышла на божественный уровень. Зачем им возвращаться в прошлое и помогать самим же себе? Решили из гуманных соображений облегчить себе этот процесс? Или это просто подачка для чуханов, что бы посмотреть, как будут барахтаться людишки в этой ветке пятого измерения (рассчитывают что они станут соседями и потом будут заглядывать друг другу в гости через гравитацию?).
Аноним 31/10/15 Суб 04:24:01 #321 №224667 
>>224665
Норкоман, если они свою жопу в прошлом не спасут, но никакого будущего у них не будет. Петля времени типа, или как эта поебень называется.
Аноним 31/10/15 Суб 04:27:43 #322 №224669 
>>224667
>но никакого
>то никакого
молниеносный фикс
Аноним 31/10/15 Суб 04:28:48 #323 №224670 DELETED
>>224651
На парашу, там все твои друзья.
>>224662
Тот, в котором мы ебали твою мамку.
Аноним 31/10/15 Суб 04:30:06 #324 №224673 
>>224665
В принципе, хоть и достаточно сложно математически, но можно теоретически построить, например, пространство-время (5+1)+(5+1). В нем, при путешествиях назад во времени, парадоксов не возникает.

Ну а в твоём "Интерстеллараре" - хуй знает. Спроси сценаристов.
Аноним 31/10/15 Суб 04:57:05 #325 №224676 
>>224665
>как-то пережило эту угрозу и вышла на божественный уровень
Амелия Бренд, попав на планету Эдмундс, имея на борту План Б, возраждает цивилизацию, (при этом все, кто жил на Земле померли) цивилизация допетривает до тайн мироздания, делает червоточину в прошлом, запускает Купера сотоварищи, и устраивает им анал-карнавал в системе Гаргантюа, Купер ныряет в ЧД, предает дочери послания, земляне съебывают с земляшки, спиздив сраный тубус.

Как-то так, я думаю.
Аноним 31/10/15 Суб 04:59:57 #326 №224677 
>>224319
Картинка еще больше вопросов оставляет, почему тогда не видно обратной стороны Луны?
Аноним 31/10/15 Суб 05:17:32 #327 №224680 
>>224394
Я-то знаю. 4>3. А ты чего так порвался?
Аноним 31/10/15 Суб 05:22:44 #328 №224681 
>>224677
>почему тогда не видно обратной стороны Луны?
Потому что обращение Луны вокруг Земли и вращение Луны вокруг собственной оси синхронизировано.
Аноним 31/10/15 Суб 05:24:49 #329 №224682 DELETED
>>224670
Какой содержательный текст. Но ты не привел номер поста.
Аноним 31/10/15 Суб 05:28:00 #330 №224683 
>>224681
Окей.
Аноним 31/10/15 Суб 05:40:43 #331 №224684 
14462592439070.jpg
14462592439081.jpg
14462592439092.jpg
Почему поверхность Титана и Венеры так похожи?
Аноним 31/10/15 Суб 06:02:06 #332 №224686 
>>224383
Пионеры и Вояджеры были пнуты под сраку Атласами и Титанами. Мирный космос поклониться должен в ноги еще за них военным.
>>224393
Кассини - Титан - МБР
Dawn - Дельта.
Ну Дельта - мирный трактор, созданный ой, на базе Тора, который вылез и Программы 437.
New Horizons снова Атлас от военщины, да еще эти проклятые оружейние бароны из Локхид-Мартин, что же такое-то!?
SOHО? Нет, опять Атлас. Тьфу ты.
Аноним 31/10/15 Суб 06:05:25 #333 №224687 
>>224684
Вообще-то непохожи.
Вот поверхность Луны похожа на поверхность Цереры, например.
Аноним 31/10/15 Суб 06:09:54 #334 №224688 DELETED
>>224682
Я номера ваших охуительных порашных постов себе не записываю. А тебе зачем? Хочешь пожаловаться на совок, но быть оригинальным?
Аноним 31/10/15 Суб 06:13:47 #335 №224690 
>>224686
МБР "Земля-космос". В мире таких полно.
Аноним 31/10/15 Суб 06:15:00 #336 №224691 DELETED
>>224688
Т.е. по существу сказать ничего не можешь.
Аноним 31/10/15 Суб 06:20:16 #337 №224692 DELETED
>>224691
По существу - с жалобами на совок пиздуй в другой раздел.
Аноним 31/10/15 Суб 06:27:28 #338 №224694 DELETED
>>224692
Ты еще и тупой. Какой мой пост тебе ьак не понравился? Тут записывать не нужно: тред перед тобой.
Аноним 31/10/15 Суб 06:33:03 #339 №224695 DELETED
>>224694
В любом обсуждении совковой космической техники периодически кто-нибудь ноет про то, как в союзе было плохо, как-будто кому-то не похуй. Не ИТТ, а на борде вообще. А теперь объясни, что же тебя так обеспокоило и задело во фразе
>Или сейчас будет очередной спич про то, какой совок был плохой?
что ты прям не можешь усидеть на месте?
Аноним 31/10/15 Суб 06:57:32 #340 №224699 DELETED
>>224695
Наверное то, что спича про совок не последовало? Да, кстати, он (такой распрекрасный) - развалился.

Ну а теперь, когда я сделал тебе одолжение, и ответил на твой вопрос, может, всё же, ответишь и на мой?
>>224662
Аноним 31/10/15 Суб 07:06:55 #341 №224701 DELETED
>>224699
Нет, не отвечу.
Аноним 31/10/15 Суб 08:00:49 #342 №224705 
>>224354
Да. Люди, которые даже не представляют себе работу по поддержке автономной станции в космосе, считают, что им будет "охуенно". Потом эти люди предлагают национализировать Эппл и еще двадцать ведущих предприятий по производству железа, перенаправить работников на работу в области освоения космоса, а решения для миллионов китайцев, занятых сейчас на сборке, у них нет. Собсно на китайцев им похуй, пусть хоть все там с голоду передохнут, ведь вопрос всечеловеческого масштаба решается! Ведь они думают, что флажок и десятка тысяч квадратных метров помещений на каком-нибудь галилеевом спутники стоит того, чтобы устроить гуманитарную катастрофу на Земле.
А еще самое печальное, что вы свои больные фантазии здесь выплескиваете, а не в /sf.
Аноним 31/10/15 Суб 08:09:15 #343 №224706 DELETED
>>224371
>дохуя (или хотя бы равно) эффективны вкладу военных технологий в фундаментальную науку
Так за что ты ратуешь, когда выписываешь очередной наезд на айфон? За новую войнушку? Очевидно, что в условиях даже холодного противостояния прогресс движется ощутимо быстрее, однако выбрали меньшее из зол с соответствующими отрицательными сторонами. Тебе очередной замес "стенка на стенку" нужен? Если так, то надеюсь, что ты себя хотя бы не в роли генерала видишь.
Аноним 31/10/15 Суб 08:34:46 #344 №224713 DELETED
>>224705
Сколько фантазий. Есть такие волшебные слова: "субсидии" и "гранты". И ничего не придется перенаправлять, ты сам максимализмом видимо еще не переболел.
>>224706
Я ратую за то, чтобы в этом итт некто не делал утверждений типа >>224235
>Хотя те же "айфоны" (во всех проявлениях) дали космосу больше, чем вся гонка вооружений.
Потому что это откровенная хуйня. Я писал, что не милитарист, о войне с жуками не мечтаю, но факты говорят именно о том, что военные дали нам большинство космических технологий. Впрочем если это все тот же анон - сходи нахуй, на вопрос о трех достижениях мирного космоса еле-еле выдавил из себя исследования дальнего космоса и слился жи
Аноним 31/10/15 Суб 09:00:59 #345 №224715 DELETED
>>224713
>"субсидии" и "гранты"
>колония на галилеевом спутнике.
Так мы про ближайшие 50 или 200 лет говорим?
И скажи уже наконец. Ты за новую холодную или даже горячую войну, как за единственный метод прогресса.
Аноним 31/10/15 Суб 09:12:40 #346 №224717 DELETED
>>224715
Ну так субсидии надо больше, тут хотя бы готовность общеста на что-нибудь, кроме новых айфонов надо бы. Я против войны, но и против общества потребления. И то и другое формы рака, выбирать чего я больше хочу, карциному или саркому я не стану. Технократия - было бы в самый раз.
Аноним 31/10/15 Суб 09:13:38 #347 №224718 DELETED
>>224717
большие, общества
быстрофикс
Аноним 31/10/15 Суб 10:03:20 #348 №224722 DELETED
>>224717
Но ведь рак не дает человеку выбор, если начинает развиваться. Остается только принимать последствия. Выражая свое "фи" айфонам ты ничего не добьешься.
>Технократия - было бы в самый раз
Как ты себе представляешь ее реализацию на данный момент?
Аноним 31/10/15 Суб 10:16:30 #349 №224724 DELETED
>>224722
От него можно вылечиться, если не слишком поздно.
>Как ты себе представляешь ее реализацию
Экономикой рулят экономисты, научной областью - ученые... Звучит дико, понимаю.
Аноним 31/10/15 Суб 12:46:08 #350 №224737 DELETED
>>224724
Ну вылечиться от него можно с тем же успехом, с каким можно ждать волшебной ремиссии. А твое рулят экономикой/наукой звучит как план "все приходят на работу и работают". Ты все ждешь, что придет какая-то горящая идеей партия, за которую проголосуют достаточное количество избирателей. Партия будет иметь большинство в думе и двигать законопроекты в интересах науки. При этом президент будет также к науке не равнодушен, и будет подписывать эти законы, а сотни чинуш на пути бюджетных денег будут свято чтить их неприкосновенность. Даже если не брать Россию, один хер нигде это сейчас нереализуемо и еще слишком много подготовительных этапов пройти предстоит.
Вот и звучит твоя технократия как утопия похлеще коммунизма. Кроме хотелок то есть что-нибудь.
Аноним 31/10/15 Суб 12:55:40 #351 №224739 DELETED
>>224737
Ты спросил как я вижу реализацию, так да, это может быть партия, может быть даже некий "мягкий" тоталитаризм, да хоть просвященная монархия, лол. Основной затык не в строе/президенте, да и при таком строе чинуши-то будут компетентные, так что и не в них дело. Как уже считай век продвигалось общество потребления, где главная задача человека больше сожрать и больше оттрахать, так надо подготовить массовое сознание к тому, что ресурсы надо тратить с умом и иногда лучше себя ограничить, чтобы следующие поколения жили в уютном мирке на Каллисто. Сейчас с развитием альтернативной энергетики, "зеленых" технологий и энергосбережения первые шаги сделаны, но их надо двигать дальше, больше и т.д. А РФ в этом плане гиблое место, не стоит ориентироваться на сырьевую республику третьего мира (аналогия с банановой республикой, если не понятно). Пока в РФ шансов на что-то подобное нет.
Аноним 31/10/15 Суб 13:15:32 #352 №224744 
>>224676
>Амелия Бренд, попав на планету Эдмундс

Что бы Амелия попала на Эдмундс она должны была пролететь сквозь чревоточину. Но чревоточину уже создали до неё. Что то тут не сходится.
Аноним 31/10/15 Суб 13:17:59 #353 №224745 
>>224744
Там банальная временная петля. ГГ сделал что-то потому, что позвонил себе в прошлое и попросил сделать это.
Аноним 31/10/15 Суб 13:19:45 #354 №224746 
>>224676
>>224744
Купер летит в червоточину > передаёт инфу про сингулярность или что там > его дочь решает то уравнение > люди сьёбывают с земли на космических кораблях > через неопределённый период времени люди создают червоточину > Купер летит в червоточину.
Как-то так вроде.
Аноним 31/10/15 Суб 13:39:33 #355 №224748 DELETED
>>224739
Главная проблема - кукарекинг. На вопрос о реализации ты отвечаешь тем, как ты это видишь. А нужно отвечать тем, что ты можешь. Нет, можно даже так - кого ты знаешь, кто может. Если не сам, то знаешь ты человека, который может и готов двигаться в этом направлении, можешь ты ему чем-то помочь?
Просто все эти наезды на общество потребления, которое может быть и далеко от идеала, но тем не менее проверено временем, со стороны людей у которых даже организованной альтернативы нет, кроме собственного манямирка, выглядят сплошной профанацией.
Главное шоб текущий порядок в труху, а новый там как-нибудь сам организуется. И все пустой треп, никаких подвижек потому что "в рашке не вариант, просто не повезло родится в нормальной стране".
Аноним 31/10/15 Суб 15:30:57 #356 №224767 
>>224690
Да, Р-7 одна из них, что по делу сказать хотел, умник?
Аноним 31/10/15 Суб 15:58:45 #357 №224778 
В порашу, суки. В порашу.
Хотя чего я хочу, спейсач же деградировал, и полнится быдлом школьным.
Аноним 31/10/15 Суб 19:24:33 #358 №224832 DELETED
>>224748
Да-да, хочешь на марс, почему не построил корабль и не организовал экспедицию, ратуешь за технократию, почему не сделал ее хотя бы в масштабе отдельной страны. У харкача странные представления, как и что в мире меняется. Рабовладельчество и феодализм тоже проверенные временем системы и работали подольше твоего общества потребления. Да и работает оно странно. Мы можем уже лет 30-40 летать к другим планетам - а не летаем. Может всем миром строить научные станции в космосе, но делаем это не так часто и не так охотно. Хорошо хоть с мертвой точки все сдвинулось, уже и китай вон хочет поселение на луне.
>>224778
Кому чего, а вшивому политика, быдло, школьники и деградация.
Аноним 31/10/15 Суб 20:23:01 #359 №224843 
Чему равна скорость гравитации? Платиновый вопрос наверное.
Аноним 31/10/15 Суб 20:27:12 #360 №224844 
>>224843
Все наблюдения и эксперименты пока утверждают, что со скоростью света.
Аноним 31/10/15 Суб 20:33:30 #361 №224845 
Где почитать о впечатлениях космонавтов роскосого о полётах на Шатолах? Как ни странно, не гуглится.
Аноним 31/10/15 Суб 20:33:56 #362 №224846 
>>224844
Ебанулся? Мгновенно.
Аноним 31/10/15 Суб 21:21:04 #363 №224853 
>>224846
И тут ты такой с пруфами на мнгновенность.
Аноним 31/10/15 Суб 22:17:41 #364 №224868 
14463190612680.jpg
1. Около Венеры радиоактивная угроза от Солнца сильнее, чем у Земли, или же расстояние между второй и третьей планетами не такое большое чтобы существенно снизить эффекты от радиации Солнца?

2. Никаких прорывов в области защиты от радиации нет на сегодня? Не появилось каких-нибудь новых материалов, которые могут защищать надежнее?

3. Корпус ПТК НП хотят сделать из углепластика, а как он от радиации защищает? Нельзя создать новое соединение углерода, со свойствами защиты от радиации?
Аноним 31/10/15 Суб 22:29:50 #365 №224870 
>>224868
> которые могут защищать надежнее?
H2O
Аноним 01/11/15 Вск 00:37:18 #366 №224896 
>>224868
Да, у Венеры радиация сильнее. Но на месячную пилотируемую экспидицию норм. А вот по году как на МКС у Земли - уже немного пиздец.
Аноним 01/11/15 Вск 01:28:48 #367 №224914 
>>224868
с защитой от радиации (гамма) принцип примерно такой: килограмм свинца защищает примерно также как килограмм воды, кремния и тд.
Те основной посыл углепластиковых конструкций идет в разрез с защитой. От химии соединений в общем это зависит практически никак. Это все со своими оговорками, но в первом приближении примерно так.
Решение: надо увеличивать сами крафты, попутно это закроет вопрос.
Интересный момент это изотопный состав материалов, какието изотопы лучше поглощают нейтроны к примеру, какие то хуже; также могут быть преимущества с точки зрения вторичной радиоактивности.
Аноним 01/11/15 Вск 04:16:01 #368 №224939 
>>223277 (OP)
Как выравнивают температуру и давление на МКС? Выход астронавтов осуществляется только на солнечной стороне или когда Земля дает тень?
Аноним 01/11/15 Вск 04:57:57 #369 №224944 
14463430776630.jpg
смотрите, тоесть гравитация - это есть искривление пространства. так. тоесть земля с большой массой прогибает пространство, и в неё всё скатывается, вода, люди, совы. чем больше масса, тем больше искривление. и от сильного искривления пространство вообще рвется, как в черной дыре, сингулярность образуется. я правильно понимаю.
Аноним 01/11/15 Вск 05:08:33 #370 №224945 
>>224944
>пространство вообще рвется
Там вообще хуй знает что. По уравнению ТО там получается, что кривизна пространства бесконечна. Это значит простую вещь - там хуй знает что и ТО не применима. То ли ТО не совершенна, то ли там привычные законы работают иначе. А скорее всего и то и другое. Рвется ли там все, или мнется, или окукливается - не известно.
Аноним 01/11/15 Вск 08:58:41 #371 №224958 
14463575214040.jpg
>>224939
Есть система термоконтроля жилых отсеков, она при помощи трубок с хладагентом отводит тепло к радиаторам (вот эти белые гармошки промеж солнечных батарей и на жилых отсеках). Американский сегмент использует воду в качестве хладагента в контуре жилой части и аммиак на фермах, российский использует что-то более мудреное и инертное.

Время ВКД стараются подбирать так, чтобы вход и выход были на освещенной стороне, а также чтобы длительность работы при солнечном свете была максимальной. Но вообще ВКД может проводиться и в тени, и проводится, т.к. длина сеанса превышает время инсоляции.
Аноним 01/11/15 Вск 09:03:44 #372 №224959 DELETED
>>224832
Дело внезапно не в харкачерах, а в присущей всем людям инертности мышления. Смене феодализма и рабовладения соответствовали глобальные события, причем географического толка. Ты же на ровном месте хочешь революцию. Скорее случится все может как раз наоборот, сначала начнется колонизация солнечной системы, а уж потом может начнут появляться предпосылки для технократии на Земле. Но знаешь, я могу ошибаться, в виду зашоренности и повышенной инертности мышления в силу возраста и жизненных обстоятельств. Однако если хочешь что-то изменить, то нужно что-то сделать, от одного хотения вклада нет. А до тех пор это сплошной кукарекинг, на который любая мимикрирующая потреблядь вроде меня будет смотреть свысока и писать про юношеский максимализм и идеализм.
Аноним 01/11/15 Вск 10:52:01 #373 №224968 DELETED
>>224959
Революцию, внезапно, не хочу совершенно. Про изменения обстоятельств согласен, но ведь мы не можем сказать без ретроспективы, что они уже не наступили - эти обстоятельства. Может быть в учебниках истории для покидающих солнечную систему через 200 лет напишут: "начало 21 века ознаменовалось быстрым расселением человека по солнечной системе, хотя этот процесс поначалу и тормозило т.н. "Общество потребления", но уже на рубеже 20 и 21 века было очевидно, что оно уступает свои позиции... предпосылками к тому..." и т.д. Т.е. те же феодалы тоже могли не понимать, что их феодам стоять осталось всего ничего. А при нынешних темпах развития изменения могут проходить буквально при жизни 1-2 поколений. А про кукарекинг - с него все и начинается, сначала Карл Маркс пишет книжку и разводится кукарекинг, а потом, спустя сколько-то лет, начинаются изменения.
>мимикрирующая потреблядь вроде меня
Зачем? Мне вот жрать, спать и покупать фордфокус в кредит не интересно. 30 лет, сейчас кантуюсь в европках, трактор потихоньку пытаюсь завести. Жить в глуши, где тихо, спокойно, чистый воздух.
Создать базу на орбите из... Ерохин 01/11/15 Вск 12:33:51 #374 №224987 
14463704312590.jpg
Саб космонёрды давно уже обдумываю идею создания базы на орбите этой планеты, до приклеетед мне как от индокитая до луны пехом, но в планах именно это и хотет. В планах разработка ракетки с запасом топлива и механизмом как гугловский дерьмо сборщик, собсна из это дерьма как раз и будет собираться база отдыха орбита-хуита. Автобот нужен для сборки помещений так же планировал запустить его не с земли а с орбиты на гелеевых шарах а потом уже запуск ракеты и "к звуздам" Какие подводные камни антошки? Что я могу упустить?
Аноним 01/11/15 Вск 12:42:36 #375 №224990 
>>224987
> на гелеевых шарах
> с орбиты
> Что я могу упустить?
Кратко - всё.
Поиграй в КСП хотя бы.
Аноним 01/11/15 Вск 12:44:12 #376 №224991 
>>224987
если кто нипонил вокруг земли куча дерьма летает из спутников-хуютников "сковорода-1" из этих как раз и спутников хотет сделать "поглащатель-3000" чтоб он всё сжирал и превращал в базу. Естессно там не будет килслого для жизни, но до этого мне как то похуй пока пусть появиться база, а там будем думать как закачать туда кислого.
Ерохин 01/11/15 Вск 12:45:25 #377 №224992 
>>224990
в ксп нет гелевых шаров для запуска со стратосферы.
Аноним 01/11/15 Вск 12:49:49 #378 №224993 
>>224992
Зато там орбита есть.
Аноним 01/11/15 Вск 12:50:46 #379 №224994 
14463714462870.jpg
>>224987
Как я понял есть вариант ловить мусор сеткой металлической, но я так понимаю закон физики о сохранении энергии ни где не отключается галочкой, так шта инерцию как гасить?
Аноним 01/11/15 Вск 12:52:38 #380 №224995 
14463715583470.jpg
>>224993
Там нет кучи говна на орбите, чтоб эксперимент по ловле проводить
Аноним 01/11/15 Вск 13:00:21 #381 №224996 
>>224987
Робятки есть ли годная литература о запуске ракетки со стратосферы. В моей голове теории это должно быть дешевле не?
Аноним 01/11/15 Вск 13:13:59 #382 №224999 
14463728394120.jpg
>>224987
В теории энергию думаю брать из солнышка ибо откуда еще блять? Из мусора робот должен варить конструкцию, но для этого нужно будет кудато накапливать липездричество видимо аккумулятор. И это проблема, т.к. он стареет и херится и после вероятно года двух его нужно заменить есть ли вариант аналога?
Аноним 01/11/15 Вск 13:14:19 #383 №225000 DELETED
>>224968
заведется жена - выкинешь весь этот инфантилизм из головы
Аноним 01/11/15 Вск 13:19:55 #384 №225001 DELETED
>>225000
Уже есть и мне как то похуй на это. Годного чего на советуй, жена.
Аноним 01/11/15 Вск 13:31:26 #385 №225002 
>>224987
С мелким мусором что делать? переплавлять? как тогда температуру нагнетать кислого то нет в этих ваших вакуумах есть вариант магнитной плавки как тогда температуру сдержать в космосах прохладненько и дует местами в тени -100 на солнышке +160 как быть?
Аноним 01/11/15 Вск 13:39:53 #386 №225003 
>>224987
Пока что считаю основной проблемы накопление энергии что на советуете? Для добычи кроме батарей солнечных думаю элемент пельтье с одной стороны солнышко с другой закрытый кожух теплоотвода как думаете прокатит не?
Аноним 01/11/15 Вск 13:44:00 #387 №225005 
>>225003
Ты не ответил, как ты собрался свое говно на орбиту вывести, для низкой орбиты надо скорость ~8 км/с, потом для твоего сборщика потребуется топливо, откуда собрался брать? Ты покурил водки и тебя осенила гениальная идея?
Аноним 01/11/15 Вск 13:49:32 #388 №225008 
>>225003
> считаю основной проблемы
Система моховиков-гироскопов на могнитной подвеске + двигатель-генератор для ввода-вывода энергии.
Причем энергия на поворот станции может быть забрана без использования двигателя, только с помощью подвески.

Это давно на свете известно, однако пока все равно фантастика уровня /завтрашний день
Аноним 01/11/15 Вск 14:05:18 #389 №225011 
>>224994
Придут дяденьки из наса и дадут тебе пизды если их спутник поймаешь, или замаскированный под пылесос\мусор спутник шпион ФСБ\КГБ\ЦРУ.
Аноним 01/11/15 Вск 14:06:08 #390 №225012 
>>224994
>Как я понял есть вариант ловить мусор сеткой металлической,
Идея неплохая сама себе, но учти что там 99% радиоактивного говна.
Аноним 01/11/15 Вск 14:16:22 #391 №225014 
>>225005
Ты похоже в читать не способен. Расписал же все. Топлив разных дохуя на цвет и вкус выбирай сиди. Запуск со стратосферы ибо в теории так дешевле, но топливка оставлю как с земли запускать на всякий-хуякий. Топливо для сборщика не нужно, потомучто иди нахуй вот почему. Энергию солнечную буду использовать для работы, а в тени накопленную.
Аноним 01/11/15 Вск 14:20:48 #392 №225016 
>>225008
Этот вариант мне известен из игрушек еще думаю его тоже реализовать
Аноним 01/11/15 Вск 14:27:46 #393 №225020 
>>225011
На нем не написанно моя фамилия и где меня искать а подключаться я к нему буду не чаще чем раз в год да и этот вариант как бэ я не рассматриваю чть более чем полностью
>>225012
согласен что радиоактивного гавна много по этому то я могу его использовать и для добычи энергии, а ежели ты про метал что в космосах то есть же способы избавления от радиаций не?
Аноним 01/11/15 Вск 14:30:29 #394 №225022 
>>225008
>моховиков
Ну это вообще орбитальная пушка.
Аноним 01/11/15 Вск 14:31:42 #395 №225023 DELETED
>>224968
>но уже на рубеже 20 и 21 века было очевидно
В том-то и дело, что никому не очевидно, кроме "ренегатов от науки". Сейчас только малые подвижки происходят по сокращению 40-часовой недели на политическом уровне. И ничего другого в сторону технократии. Люди хотят айфоны и не хотят в космос. А вообще эппл - частная корпорация, исправно платящая налоги. Почему именно айфон выбрали врагом прогресса - не понятно. Враг прогресса - растраты бюджеты на социальные программы и нужды. В холодную войну бюджетные деньги просто вагонами шли в программу Аполлон у американцев. Были идейные борцы с коммуняками. А сейчас? Каждый хочет обласкать черное меньшинство, обещает бесплатную медицину, субсидии на развитие кварталов меньшинств, программы на учебу для меньшинств, вот это все. Сюда и уходят бюджеты, которые могли бы пойти на космические программы. Так что же теперь, бороться с гуманистическим подходом в политике? Не по причине ли этого подхода ты лыжи в европку навострил?
Аноним 01/11/15 Вск 14:53:55 #396 №225028 
>>225020
Запеленговать тебя смогут и баз имени и фамилии! Срсли!
Нет не можешь, сам об этом думал. В случае мусора - это скорее всего оплавленный кусок говна в свинце, который надо будет долго очищать и доводить до массы взаимодействия. к тому-же у разных спутников разные источники, разные наполнители, разные конверторы. Кто-то на уране работает, кто-то на плутонии, у кого-то термодрайверстоит у кого-то пьезо кристалл.
Короче - это жопа.
Аноним 01/11/15 Вск 14:58:29 #397 №225031 
>>225014
> Топливо для сборщика не нужно, потомучто иди нахуй вот почему.
Ты собираешься сидеть на месте и ждать, пока к тебе прилетит металлолом? Хорошая мысль, может лет через пару тысяч сможешь набрать на собачью конуру.
Аноним 01/11/15 Вск 14:59:34 #398 №225033 
>>225020
>На нем не написанно моя фамилия и где меня искать а подключаться я к нему буду не чаще чем раз в год
Если ты спиздишь оборудование ценою в десятки миллионов доллларов - тебя найдут.
Аноним 01/11/15 Вск 15:24:01 #399 №225035 
>>225014
Т.е. Ты на шарах решил поднять РН с запасом топлива? Охуенная мысль, почему в наса/роскосмосе/спасеиксе никто еще не придумал? Ангара поди в 1,5 раза легче выйдет, если ее на шарах в стратосферу поднять! Ок, предположим ты таки стартовал с земли и вышел на НОО шары твои полная хуйня, ты ведь понимаешь, если не дебил, дальше тебе потребуется постоянно топливо чтобы на орбите держаться, для маневров (в том числе уклонения, т.к. если твоя скорость 8 км/с и на встречу летит обломок с 8 км/с, то на 16 км/с ты нихуя кроме пиздюлей не поймаешь, а еще на догонялки с обломками. Пока не научишься топливо клепать из света и хотелок - твой проект хуета. Тем более что хлам уж тогда лучше в атмосфере сжигать, чем выводить на высокие орбиты (многодельты) и клепать из них станцию. А если не выводить - станция у тебя пизданется витков через 200, или надо будет постоянно ее разгонять.
Аноним 01/11/15 Вск 15:31:41 #400 №225036 DELETED
>>225023
Я тебе писал, что сейчас такие изменения могут уже быть, просто без ретроспективы ты хуй их заметишь.
>айфон выбрали врагом прогресса
Не врагом прогресса, а воплощением меркантильных хотелок, когда каждые пару лет ничего не меняя продают гору одинакового говна и все радостно стоят сутками в очереди его купить.
Про соцсферу не могу согласиться,тем более, что ты смешал в кучу и политические ходы (меньшинства) и соцсферу в целом (что как раз действует позитивно на общество. Медицину надо развивать, делать доступной, а вот облизывать каждого "нитакого как все" - не надо.
>лыжи в европку навострил
Как представлю выход на пенсию в РФ, сразу хочу тт и патрон хотя о чем я, скакалку и обмылок.
Аноним 01/11/15 Вск 16:21:24 #401 №225046 
>>225014
>Запуск со стратосферы ибо в теории так дешевле
зависит от того как ты крафт в стратосферу доставлять будешь, там выхлоп у ракеты будь здоров, а бронированную ёбу в стратосферу заебешься тащить.
в пизду, проще долить лишних сто тонн топлива
Аноним 01/11/15 Вск 16:24:12 #402 №225048 
>>224423
>>223280
>223280
>>223280

>гравитационные волны

С этого момента поподробнее, пожалуёста.

Мне применительно к достижению орбиты этой планеты без достижения орбитальных скоростей путём тупого выстреливания реактивным снарядом с приготовлением отбивной из астронавтов, а плавным ускорением, не превышающим 2-3 ЖЭ. Ну космос для всех, ну ты понел, ну и вот.
Аноним 01/11/15 Вск 17:27:52 #403 №225067 
>>223277 (OP)
Не нашел треда умных вопросов, спрошу здесь:

Что такое пытается скрыть наса от нас, из-за чего не направляют хаббл на луну?
Аноним 01/11/15 Вск 17:30:14 #404 №225068 
>>225067
>хаббл
>на луну
Чё?
Аноним 01/11/15 Вск 17:33:00 #405 №225069 
>>225068
Фотачке тех мест, где по легенде прилунялися аполлоны, через плечо.
Аноним 01/11/15 Вск 17:36:36 #406 №225070 
>>225069
Ты можешь обьяснить нахуя это делать хабблу, если даже ссаный школьный телескоп может это сделать?
Аноним 01/11/15 Вск 17:37:58 #407 №225071 
>>225070
Он думает, что Хаббл сможет увидеть следы астронавтов на Луне.
Аноним 01/11/15 Вск 17:41:22 #408 №225072 
>>225071
Вот делать нехуй астрономам, кроме как рассматривать следы какого-то хуя на луне. Ну серьёзно! Это установка за овер 100 миллионов долларов. Щас её развернут на луну!
Аноним 01/11/15 Вск 17:45:48 #409 №225073 
>>225072
> Это установка за овер 100 миллионов долларов.
>>225071
> не может увидеть следы астронавтов.

Не говорите мне больше о роспилах в раскосом тогда. Будьте последовательны
Аноним 01/11/15 Вск 17:46:17 #410 №225074 
>>225072
Дело даже не в этом. Хаббл не сможет ничего там увидеть - он создан, чтобы пялиться в глубокий космос, а Луна буквально под боком. Всё равно что смотреть в телескоп на ручку, лежащую перед тобой на столе. Ты нихуя не увидишь.
Аноним 01/11/15 Вск 18:05:08 #411 №225083 
>>225074
Дурак?
Более того, и на Землю его направить можно.
Аноним 01/11/15 Вск 18:12:48 #412 №225087 
Местные дегенераты просто плохо представляют размеры Луны и её удалённость.
Аноним 01/11/15 Вск 18:14:15 #413 №225088 
>>225069
Разрешающая способность Хаббла недостаточна, что бы разглядеть там какие-либо следы.
>скрыть наса
НАСА скрывает от кого-то что-то только в фантазиях обитателей зогача. LRO давно снял все места посадки лунный миссий.
Аноним 01/11/15 Вск 18:14:43 #414 №225089 
>>225087
Это не местные.
Аноним 01/11/15 Вск 18:21:39 #415 №225093 
14463912992860.jpg
>>225074
Я даже больше тебе скажу, Хаббл таки фоткалл Луну, прикреплейтед.

Как видно, ничего не видно. Только кратерус Коперникус, и это неспроста.
Из-за скорости Хаббла и скорости Луны, снимки могут быть только такого вот качества.
Здесь суперкороткая выдержка, благо яркость Луны это позволяет, и толстое гидирование тупо гироскопами.

Бида в том, что его точнейший интерферометрический гид совмещен с главной осью и в качестве гидирующего объекта может использовать только точечный источник, звезду.

Как-то так.
Аноним 01/11/15 Вск 18:25:32 #416 №225095 
>>225093
Так а хули контраст подкрутить и навесить на кратер какой яркий, он же не картинку с камеры юзает, кек.
Аноним 01/11/15 Вск 18:30:28 #417 №225100 
14463918280890.jpg
Есть ли где материалы по изучению линкоса? Гугл отмалчивается, решил спросить здесь.
sageАноним 01/11/15 Вск 19:32:48 #418 №225118 DELETED
>>225100
Анус свой изучи.
Аноним 02/11/15 Пнд 03:34:20 #419 №225193 
>>225100
Тебе нужна книга Lincos: Design of a Language for Cosmic Intercourse. Part 1, если погуглить основательно, она есть в интернетах.
Аноним 02/11/15 Пнд 07:36:48 #420 №225220 
>>225020
>я могу его использовать и для добычи энергии
Ээм, каким образом?
Аноним 02/11/15 Пнд 09:05:42 #421 №225223 
>>225220
раскатает в листы, покрасит черной краской, поставит стирлинг и будет делать инергию.
Аноним 02/11/15 Пнд 10:30:27 #422 №225239 
>>224589
Сейчас видео мне недоступны, но показывали на мкс, как в невесомости крутились космонавты чтоб двигаться вперед.
Аноним 02/11/15 Пнд 13:09:50 #423 №225250 
14464589901130.jpg
14464589901141.jpg
14464589901142.jpg
Спейсаны, поясните. Куда девается генераторный газ на второй ступени F9 я вижу. Как он горит на первой ступени F1 я тоже вижу. Вопрос: куда он утилизируется из 9 двигателей первой ступени F9?
Аноним 02/11/15 Пнд 13:13:13 #424 №225251 
>>224585
Было в ЮТ. Нужны ласты и плавник для стабилизации.
Аноним 02/11/15 Пнд 13:14:55 #425 №225252 
14464592960160.png
>>225250
Хуёвая первая картинка, эта лучше.
быстрофикс
Аноним 02/11/15 Пнд 13:47:51 #426 №225253 
test
Аноним 02/11/15 Пнд 19:01:15 #427 №225288 
[hide]test[/hide]
Аноним 02/11/15 Пнд 19:02:39 #428 №225289 
<hide>test</hide>
Аноним 02/11/15 Пнд 19:03:09 #429 №225290 
<b>test</b>
Аноним 02/11/15 Пнд 19:14:06 #430 №225292 
>>225288
>>225289
>>225290
Держи, болезный. https://2ch.hk/faq.html
Аноним 02/11/15 Пнд 22:42:06 #431 №225328 
>>225035
Но ведь весь хлам летает в одну сторону. Как и говноспутник этого довинчи.
Аноним 02/11/15 Пнд 23:12:18 #432 №225333 
14464951382920.jpg
Поясните, эта обоина - просто набор рандомных точек или стилизованный вид неба с созвездиями?
Аноним 02/11/15 Пнд 23:54:13 #433 №225340 
>>225328
>Но ведь весь хлам летает в одну сторону.
http://stuffin.space
В одну, конечно. И говноспутник этого деятеля тоже должен летать по разному, если хочет что-то собрать. А с большим наклонением встречным курсом много летает. А если без маневров - он 3 обломка за 100 лет соберет.
Аноним 03/11/15 Втр 01:33:41 #434 №225361 
>>225340
а какже эти которые кричат, орбита просрана засрана, не увидит венера милочка своего, мы все в ловушке и тд
Аноним 03/11/15 Втр 01:42:18 #435 №225365 
>>225361
Так сам посмотри по ссылке, а еще кучи мелкого мусора. Просто космос большой и даже на низкой орбите это километры, тысячи их. "Хорошие" орбиты засраны и активно засираются, плюс даже мелкий объект может представлять опасность, особенно когда в космос начнем выходить не по праздникам.
Аноним 03/11/15 Втр 02:18:04 #436 №225386 
14465062841210.png
сфотал из плоскости ГСО. Но что это за вторая плоскость, менее выраженная и хрень между ними?
Аноним 03/11/15 Втр 02:29:51 #437 №225391 
>>225386
Есть осторожное подозрение, что пуски с Байконура.
Аноним 03/11/15 Втр 03:17:13 #438 №225408 
>>225386
Влияние гравитации Луны.
Аноним 03/11/15 Втр 05:54:24 #439 №225431 
Наблюдаю в окно две яркие точки в небе. Какие планеты должно быть видно в это время года? Это Венера и Марс, или Венера и Юпитер7
Аноним 03/11/15 Втр 06:34:59 #440 №225432 
>>225431
>Какие планеты должно быть видно в это время года?
Наверное все.
Аноним 03/11/15 Втр 06:47:25 #441 №225433 
>>225431
>>223652
Аноним 03/11/15 Втр 08:24:07 #442 №225441 
>>225333
Хуй знеает что это. Почти все точки одинаковой яркости, цвета и квадратные. Хуйня какая-то это.
Аноним 03/11/15 Втр 09:16:59 #443 №225442 
>>225431
Самые яркие Венера, Юпитер. Марс более тусклый.
Аноним 03/11/15 Втр 10:24:56 #444 №225448 
Где взять космические обои?
Но не обычную художественную, а реально крутые?
Аноним 03/11/15 Втр 10:42:12 #445 №225449 
>>225448
https://2ch.hk/wp/
Тут спроси.
Аноним 03/11/15 Втр 14:57:35 #446 №225487 
>>225448
http://apod.nasa.gov/apod/archivepix.html
Для начала.
Аноним 03/11/15 Втр 23:32:20 #447 №225624 
14465827405360.jpg
Короче, мировые светила уровня /spc/, у меня к вам такой вот вопрос. Много где встречается упоминание о такой убер хуйне, как сфера дайсона, как я понял многие согласны с тем фактом, что ее можно построить, но меня интересует КАК? Т.е. какие технологии нужны что бы сделать такую НЕХ? Там же одной энергии для надо столько, что профит от всей этой затеи будет нулевой
Аноним 03/11/15 Втр 23:42:15 #448 №225626 
>>225624
>мировые светила уровня /spc/
Проиграл.
Сфера Дайсона потребует материалов много и нереально прочных причем на "экваторе" будет действовать сила намного больше, чем на полюсах и ее может разорвать + она неустойчива и насколько помню, у полюсов будет невесомость, где будет проябываться атмосфера (если таковая есть), энергии же дохуя от звездочки в центре же. Овердохуя. Погугли другие варианты еще, кольцо Нивена и раковину Покровского, они тоже неустойчивы и потребовали бы коррекции, но лишены недостатка с разницей напряжения материала и областей невесомости.
Аноним 03/11/15 Втр 23:49:31 #449 №225630 
>>225626
> Сфера Дайсона потребует материалов много и нереально прочных
Мне интересно, считал ли кто-нибудь, если использовать все, что есть в нашей Солнечной Системе (планеты, астероиды и пр.) и сделать из этого сферу Дайсона радиусом 1 а.е., какой толщины она выйдет? Оно-то можно и самому посчитать, но я подзаебался за день, если ни у кого нет готовых цифр - сам посчитаю завтра. Чисто интуитивно - материала не хватит нихуя даже на простую оболочку, особенно если учесть, что из водорода строить её как-то неудобно.
Аноним 03/11/15 Втр 23:53:09 #450 №225631 
>>225630
Можно строить не из материи
Аноним 03/11/15 Втр 23:54:45 #451 №225634 
>>225631
Нельзя.
Аноним 03/11/15 Втр 23:55:52 #452 №225635 
>>225634
Почему?
Аноним 04/11/15 Срд 00:01:56 #453 №225637 
14465845167250.jpg
>>225626
Я хочу понять не где ее распидорасит, а как ее вообще построить, какие механизмы для этого нужны, ну и откуда брать энергию для этого строительства.
Аноним 04/11/15 Срд 00:16:48 #454 №225642 
>>225630
Мне кажется там и в миллиметр не уложиться
Аноним 04/11/15 Срд 00:32:26 #455 №225644 
>>225642
Патамушта ты так скозал?
>>225634
Тащемта можно. Электромагнитные поля наше все. Только непонятно нахуя
Аноним 04/11/15 Срд 00:40:10 #456 №225645 
>>225624
для начала тебе требуется определиться, что ты понимаешь под сферой дайсона. Механизм-система по совершению полезной работы с использованием существенной доли энергии звезды или как этот >>225626 сразу сайфай конструкции.

>>225630
на вскидку чтото в духе полуметра(оптимистично), два газовых гиганта при этом остаются
sageАноним 04/11/15 Срд 00:45:00 #457 №225647 
>>225645
>>225644
>>225624

— Строим фантастическую ёбу!
— Строим!
— А зачем мы строим эту ёбу?
— А хуй его знает.
Аноним 04/11/15 Срд 00:45:16 #458 №225648 
14465871162760.png
Как это можно в полметра раскатать?
Аноним 04/11/15 Срд 00:45:25 #459 №225649 
>>225644
>Тащемта можно. Электромагнитные поля
Поле тоже является материей. Пиздуй школьные определения читать.
Аноним 04/11/15 Срд 01:00:10 #460 №225652 
>>225448
Эх, был тут тред в 2012-2013 c кучей реальных охуенных фотографий.

http://arhivach.org/thread/4651/

Пикчи проебаны, но есть много ссылок на оригиналы.
Аноним 04/11/15 Срд 01:04:06 #461 №225655 
>>225648
в этом нет практического смысла
>>225647
чем она фантастическая?
Аноним 04/11/15 Срд 01:04:50 #462 №225656 
>>225649
>Поле=материя
Ты из кефиротреда?
Аноним 04/11/15 Срд 01:07:55 #463 №225657 
>>225655
> чем
Бессмысленностью создания.
Аноним 04/11/15 Срд 01:10:18 #464 №225658 
>>225657
А халявная энергия от звезды, к тому же вся?
sageАноним 04/11/15 Срд 01:33:35 #465 №225661 
>>225655
>чем она фантастическая?
Нет задач.

>>225656
А ты из детского сада? Поля материальны.
Аноним 04/11/15 Срд 01:41:11 #466 №225663 
14465904715630.jpg
>>225661
Поле=материя?
sageАноним 04/11/15 Срд 01:48:45 #467 №225665 
>>225663
Не пугай меня. Не понимать, что поля являются формой материи — это сильно.
По-твоему получается, что поля материей не являются? А что они тогда? Дух святой?
Аноним 04/11/15 Срд 01:53:51 #468 №225667 
14465912318890.jpg
Хуйня какая-то, но пересчитывать я не буду. И с разметкой наверняка объебался.

-----------------------------
Масса конструкции, деленная на объем, равна плотности материала:
m/(4πr2Δr) = ρ

Объем сферы Дайсона здесь - это объем сферического слоя. Взята упрощенная формула для Δr << r.

Цифры. Возьмем ρ = 7·103 кг/м3 (плотность железа) и массу m = 0.0014 M = 2.8·1027 кг (масса всего в Солнечной системе, кроме Солнца). Радиус - 1 а.е.: r = 1.5·1011 м.

Считаем. Делим массу на плотность, получаем объем конструкции: V = 4·1023 м3. Делим его на 4πr2 (2.8·1023 м2), получаем Δr = 1.43 м.

Т.е. чё, на полутораметровой толщины сферу на 1 а.е. материала всё же набирается?
Аноним 04/11/15 Срд 01:57:50 #469 №225668 
>>225663
https://ru.wikipedia.org/wiki/Материя_(физика)#.D0.9E.D1.81.D0.BD.D0.BE.D0.B2.D0.BD.D1.8B.D0.B5_.D0.B2.D0.B8.D0.B4.D1.8B_.D0.BC.D0.B0.D1.82.D0.B5.D1.80.D0.B8.D0.B8
Аноним 04/11/15 Срд 01:57:59 #470 №225669 
14465914799050.png
>>225663
>>225665
sageАноним 04/11/15 Срд 01:58:13 #471 №225670 
>>225667
Бери тогда уж чистый углерод, а в качестве материала — фуллерит. Если уж страдать хуйней, то более научно.
Аноним 04/11/15 Срд 02:02:31 #472 №225671 
>>225667
Ну и сколько в Солнечной системе планет, состоящих полностью из чистого железа? Если мне память не изменяет Si куда более распространен
Аноним 04/11/15 Срд 02:03:51 #473 №225672 
>>225624
>>225626
наботы и кварки.хуярки. материю можно создавать из ничего.
Аноним 04/11/15 Срд 02:04:39 #474 №225673 
>>225668
>Вещество в классическом понимании. Состоит преимущественно из фермионов
Аноним 04/11/15 Срд 02:09:10 #475 №225676 
>>225667
Ты учти еще, что ты и газовые гиганты, как металлические планеты посчитал, а они основной вклад в массу дают
Аноним 04/11/15 Срд 02:23:17 #476 №225683 
>>225673
Материя ≠ вещество.
Аноним 04/11/15 Срд 02:25:31 #477 №225684 
>>225683
Материя=поле, но ≠ вещество?
ЯННП
Аноним 04/11/15 Срд 02:26:29 #478 №225685 
>>225676
Я лишь высчитывал, хватит ли вещества в СС по массе.
Аноним 04/11/15 Срд 02:27:32 #479 №225687 
>>225684
Материя имеет разные формы, в том числе вещество и поле. Это преподают на физике в школе.
sageАноним 04/11/15 Срд 02:29:52 #480 №225688 
>>225672
Укатывайся из тренда создавать материю, демиург мамкин.
Аноним 04/11/15 Срд 02:30:54 #481 №225689 
>>225657
То-то мы сейчас нефть жжем, солнечными батареями поля засеиваем, ветряками засаживаем и атом приручаем. А еще на термояд заримся. А тут вся энергия звезды, сколько там нулей будет в ваттах?
>>225637
Если мы дойдем до возможности создания подобных материалов - думаю вопросы отпадут сами. Что это былет, нанороботы, прямое управление материей, гравитационные поля или магия - уже не важно. Энергию брать от звезды, например.
Аноним 04/11/15 Срд 02:31:50 #482 №225690 
>>225689
былет=будет
почин
Аноним 04/11/15 Срд 02:33:40 #483 №225691 
>>225657
А забыл еще про поверхность. Которой хватит чтобы бесконтрольно размножаться много лет и еще останется, чтобы несогласных ссылать на лесоповалы.
Аноним 04/11/15 Срд 02:33:45 #484 №225692 
>>225684
Множество "материя" включает в себя подмножества "поле" и "вещество"
Аноним 04/11/15 Срд 02:35:49 #485 №225693 
>>225689
А куда девать избыток энергии? В космос излучать уже не получится мы же сферу построили что бы удержать излучение звезды, оно же спалит все нахуй
Аноним 04/11/15 Срд 02:38:15 #486 №225695 
>>225693
Цивилизации трех типов помнишь, наши хотелки по энергии сейчас упираются в производство оной, на этапе развития, когда построим сферу (а точнее кольцо или раковину), уже должны потреблять считай всю энергию звезды. А тепло излучать, чем не получится-то? Внешняя поверхность же есть. Грей и рассеивай.
Аноним 04/11/15 Срд 02:40:42 #487 №225696 
>>225658
>>225689
> То-то мы сейчас нефть жжем
>>225691

А как насчет жить по миллиарду лет и время от времени переселяться на экзоплонети/зажигать новые светила? Не судьба?

Звезды-то не вечны.
Аноним 04/11/15 Срд 02:41:42 #488 №225697 
>>225696
А как насчет
*ограничив размножение

мгновеннофиск
Аноним 04/11/15 Срд 02:41:49 #489 №225698 
>>225696
Это дальше, ПВК, разделение на коричневые карлики и сборка по надобности заметно круче сфер и колец.
Аноним 04/11/15 Срд 02:42:07 #490 №225699 
>>225695
>Цивилизации трех типов помнишь
Вот же ж растиражировали в совке этот кардашьяновский бред.
Аноним 04/11/15 Срд 02:42:09 #491 №225700 
>>225695
Ну чтобы нагреть внешнюю, нужно нихуево так сначала подогреться изнутри, или как ты тепло наружу будешь отдавать?
Аноним 04/11/15 Срд 02:43:42 #492 №225701 
>>225698
Главное - не это. Главное - продление разумной жизни и разумного размножения. Остальное приложится.
Аноним 04/11/15 Срд 02:45:32 #493 №225702 
>>225699
Нормальное деление, чо.
>>225700
Бери сферу Дайсона с активным охлаждением, спалишь с пассивным - по гарантии не поменяешь.
>>225701
Двачую. Но сначала все равно надо освоить звезду, прежде чем ими начать в пинг-понг играть по надобности.
Аноним 04/11/15 Срд 02:46:18 #494 №225705 
А как вообще можно сместить с орбиты планету, ну допустим размером и массой с Марс, не уебав по ней другой планетой, астероидом и т.п. ?
Аноним 04/11/15 Срд 02:47:36 #495 №225706 
>>225705
Манипулируя гравитацией, поставив на нее планетарное двигло, излучением...
Аноним 04/11/15 Срд 02:49:46 #496 №225708 
>>225706
>Манипулируя гравитацией
По подробней можно?
Аноним 04/11/15 Срд 02:51:30 #497 №225710 
>>225708
Напрямую или двигая рядом некие массивные объекты. Как? А никто не знает, а второе есть некая разновидность исходного вопроса. Хотя если окажется, что черные дыры легче двигать, чем планеты, то можно мелкой черной дырой орбиту сместить.

Мы точно не в сайфае сидим? Вопросы скорее к фантастам. жи.
sageАноним 04/11/15 Срд 02:58:54 #498 №225721 
>>225710
Ну ты-то сидишь на тяжелых наркотиках по ходу, а не на сайфае.
Аноним 04/11/15 Срд 02:59:46 #499 №225722 
14465951869820.jpg
>>225710
Хуево осознавать, что мы знаем теоретическую возможность осуществления убер-мега-чего-то, но нихуя не знаем, как это воплотить в жизнь.
Аноним 04/11/15 Срд 03:01:38 #500 №225724 
>>225721
Почему? Потому что читал его в детстве? Все эти "мир-кольцо"? Или потому что знаю, как теоретически сделать огромную электростанцию из быстровращающейся черной дыры? И вообще, чего я не так сказал?
Аноним 04/11/15 Срд 04:08:44 #501 №225755 
>>223277 (OP)
насколько поехавшим надо быть чтобы хотеть в космос
Аноним 04/11/15 Срд 06:08:44 #502 №225771 
>>223652
>>225442
добра
Аноним 04/11/15 Срд 13:52:12 #503 №225798 
>>225693
какой избыток, иди отседова, самим не хватает
космические бомжы
Аноним 04/11/15 Срд 13:58:00 #504 №225800 
>>225724
не обращай внимание, это любители видеть траву во всех кого не понимает, думать же не надо.
Аноним 04/11/15 Срд 14:14:55 #505 №225802 
>>225642
Объем, который займет сфера толщиной 50 см, закрывающая Солнце на расстоянии 1 а.е. , будет приблизительно равен объему Юпитера. Но при этом стоит учитывать, что Юпитер сжат под действием гравитации, и если его "освободить", то материи хватит и на сферу полутораметровой толщины. Но да, сфера из водорода - такое себе веселье. Нужны металлы, а металлов, которые гипотетически можно будет получить путем ядерного синтеза из юпитерианского газа, получится сильно меньше.
Аноним 04/11/15 Срд 15:12:40 #506 №225813 
14466391600490.jpg
Посоны, поясните за интерстеллар. Там была планета, на которой 1 секунда длилась как 100500 лет на земле. Разве это возможно? Ну, то есть я прекрасно знаю, что объекты, находящиеся в большом гравитационном поле или двигающиеся с околосветовой скоростью будут иметь другое восприятие времени. Но какого хуя? Они на этой планете спокойно находились, значит гравитация на ней не большая. Получается дело в орбитальной скорости этой планеты и гравитации черной дыры? Тогда какого хуя корабль, находящийся на орбите этой планеты, до спуска на неё, не испытывал такой разницы в скорости течения времени относительно земного? По идее же должно быть поебать, что на планете, что хуй знает где ещё в окрестностях черной дыры 1 секунда должна равняться 100500 годам на земле. Я чего-то не понимаю или создатели хуйню сморозили? Какого хуя 1 планета рыжая, а другая нет?
Аноним 04/11/15 Срд 15:20:22 #507 №225814 
>>225813
> Получается дело в орбитальной скорости этой планеты
This. Там 0.9 и дальше много двеяток от световой. Корабль в это время подальше висел, хотя все равно по идее должен был испытывать что-то похожее в меньшем масштабе. Я для себя списал это на упрощение и потери времени при подъеме/спуске с планеты, хотя это достаточно средненькое объяснение.
Аноним 04/11/15 Срд 15:20:27 #508 №225815 
>>225813
>Посоны, поясните за интерстеллар
Поясняю - говно. Остаток поста не читал.
Аноним 04/11/15 Срд 15:29:46 #509 №225819 
>>225814
>Там 0.9 и дальше много двеяток от световой
Планетарная масса на субсветовой скорости? Да, это твердая НФ.
Аноним 04/11/15 Срд 15:35:32 #510 №225821 
>>225813
Интерстеллар хуйня. И наука тамошняя такая же хуйня. Даже если предположить что планета находилась так близко к ЧД чтобы почувствовать сокращение времени, то:
1. Расстояние от орбиты, на которой находился корабль, до поверхности планеты, по сравнению с расстоянием до ЧД, вообще не существенно, и не было бы такой разности в возрасте с чуваком на корабле. НО ТОГДА НЕ БЫЛО БЫ ТАКОЙ ДРАММЫ!
2. Я даже не представляю какой массы должна была быть ЧД чтобы было такое огромное сокращение времени. Считать, конечно, лень, но кому интересно могут проверить. Хуйня в общем.
Аноним 04/11/15 Срд 16:27:25 #511 №225834 
>>223277 (OP)

Я тут случайно заставку для spc/ отдела запилил, стоит ли по такому случаю создавать "Космических поделок анона тред"? Или просто в подходящий тред которого нету вкинуть?
Аноним 04/11/15 Срд 16:28:57 #512 №225837 
>>225834
Был когда-то тред баннеров. Ну вкинь тут. Самый популярный тред после огурцов. Ну или создай тред, но я не рекомендую.
sageАноним 04/11/15 Срд 17:05:07 #513 №225852 
>>225724
Если уж захотелось пофантазировать, то можно делать это более "твердо", т.е. научно.
Сделать мир-кольцо из пояса астероидов и из части Юпитера намного более реально, чем эту ебучую неустойчивую сферу.
Кольцо тоже неустойчиво, но его реально стабилизировать, если уж уровень технологий позволил его построить.
Про то, что перемещать черные дыры возможно проще, чем планеты, вообще гаубица. ОТО есть, но нет, хочу, бля, чушь нести.
Аноним 04/11/15 Срд 18:28:08 #514 №225881 
>>225821
Ты еще забыл про то, что приливные силы бы не цунами вызывали, а разорвали бы к херам планету, растерли в пыль, а потом она бы красиво светилась бы в рентгеновском диапазоне в составе аккреционного диска.
>>225852
Читай мой первый пост. >>225626
А про ЧД, ты вот как можешь сравнить что проще, планеты по орбитам двигать или черную дыру? И то и то сейчас область НФ. Заодно ответь, что проще для быстрой доставки заказать - авиапочту или голубями? Гаубица у него. Когда цивилизация будет оперировать звездами и распылять планеты на сооружения у нее может довольно сильно поменяться отношение к "проще" и "труднее".
Аноним 04/11/15 Срд 18:29:36 #515 №225883 
>>225881
> Ты еще забыл про то, что приливные силы бы не цунами вызывали, а разорвали бы к херам планету, растерли в пыль, а потом она бы красиво светилась бы в рентгеновском диапазоне в составе аккреционного диска.
А теперь посчитай градиент гравитации у ГС ЧД побольше. И без циферок не возвращайся.
Аноним 04/11/15 Срд 19:17:11 #516 №225896 
>>225852
Суть сферы в поглощении энергии звезды. Суть кольца лишь в расширении жизненного пространства. Неравноценная замена так-то.
Аноним 04/11/15 Срд 19:32:56 #517 №225900 
14466547761180.png
14466547761201.png
>>225813
>Разве это возможно?
Да, возможно.
>на орбите этой планеты
Он не находился на орбите этой планеты, он находился на орбите Гаргантюа, об этом говорилось в фильме.
Аноним 04/11/15 Срд 19:55:05 #518 №225903 
>>225802
как анон правильно подметил, кремния то дохера - связывать кислород, силаны варить - такчто водород не совсем бесполезен и в принципе может повысить выход конструкционных материалов.
Аноним 04/11/15 Срд 20:05:59 #519 №225905 
14466567595020.jpg
>>225883
Посчитали все давно, дыра должна быстро вращаться, планета должна быть постоянно повернута одной стороной, для невращающейся черной дыры таки разорвало бы. Даже с учетом что она сверхмассивная. А для цунами - она должна еще и раскачиваться. С интересом почитал, как всю эту хуету объяснили с точки зрения науки, в том числе ту, о которой Нолан и не знал, снимая.
Аноним 04/11/15 Срд 20:14:44 #520 №225907 
>>225821
>Считать, конечно, лень
посчитай я немогу, будь доброаноном, за одно и градиент прикинем какая разница течения времени на 1 световой секунде.

>>225852
в чем кстати проблема неустойчивости, рассматривается самый тупой вариант вращать всю сферу, спрашивается зачем, даже в .... да в пень зачем рассматривать изначально дохлую, но не исключительно по причине неустойчивости, конструкцию. Сама конструкция, в том варианте в котором она изначально появилась, дохлая в первую очередь потому что меньше ее части не представляют собой действующие системы. Те такая сфера, в том виде как она появилась, должна быть построена целиком, и только после этого начинается профит.

Более рационально, для людей, строить по мере роста потребностей, и чтобы в пределе это образовывало механизм Сфера Дайсона II, назовем это так.
Строим коллекторы, в облаке Оорта строим хабитаты имеем цепочку: солнце-коллектор-хабитат. Облако может не очень удачное место, если там нет планетки под разбор, и если там бяда с тяжелыми элементами, но там годная температура за бортом, высокий кпд.
Но принципе вообщем примерно такой.
Фазовые скорости коллекторов на орбитах и баз в ебенях будут ессно различны придется мутить всякие хитрые алгоритмы и тд - но это можно сделать.
На начальном этапе конечно таких сложностей не требуется.

>>225896
переходи на мою сторону, к меня есть хабитаты, кольцо тоже г полное.

Аноним 04/11/15 Срд 20:42:37 #521 №225909 
>>225821
>И наука тамошняя такая же хуйня.
Двощи удивительная вещь. Вот сидит астрофизик с мировым именем, один из ведущих современных специалистов по теории гравитации, обосновывает всю хуйню на 300-х страницах, выкатил список литературы, ссылки и прочее. И тут анонимас на двачах: "Да хуйня тамошняя наука". Ну и считать и читать ему лень. Главное, что хуйня и все тут. Это пусть астрофизики считают, а насчитают если, так все равно хуйню какую-нибудь.
Аноним 04/11/15 Срд 21:21:13 #522 №225922 
>>225909
Если я настрочу бред с говноссылками на 5к страниц - не говном это не станет.
Аноним 04/11/15 Срд 21:23:50 #523 №225925 
>>225909
Да тут кое-кто и всю современную науку говном считает.
Аноним 04/11/15 Срд 21:49:35 #524 №225932 
>>225922
>Если я настрочу бред с говноссылками
В твоем случае - да, не станет.
Аноним 04/11/15 Срд 22:14:01 #525 №225939 
>>225837

Окай, будем хвастаться своими убогостями тут: >>225938 (OP) ..
Аноним 04/11/15 Срд 23:18:13 #526 №225967 
14466682932960.jpg
Допустим у нас имеется бесконечная электромагнитная катапульта, которая может разгорять магниты до охуевшей скорости. Можно ли тогда достичь световой скорости?
Аноним 04/11/15 Срд 23:24:33 #527 №225971 
>>225967
Нет.
Аноним 04/11/15 Срд 23:26:17 #528 №225973 
>>225971
Почему?
Аноним 04/11/15 Срд 23:52:40 #529 №225984 
>>225973
Потому что придет Эйнштейн и будет ебать тебя метровым хуем, пока ты не выучишь физику.

Потому что нельзя достичь скорости света, можно к ней приблизиться, но достичь - нет.
Аноним 04/11/15 Срд 23:56:06 #530 №225987 
>>225973
Тебя в гугле забанили,?
Аноним 04/11/15 Срд 23:56:59 #531 №225988 
>>225967
Да, но только если такая-то котопульта будет не закольцованной.
Аноним 05/11/15 Чтв 00:18:14 #532 №225989 
>>225925
не стоит и преувеличивать ее возможности, я тут в медач зашел и слегка прихуел со всяких стори - тоже бля наука еба
Аноним 05/11/15 Чтв 00:25:43 #533 №225992 
14466723436020.png
>>225987
не совсем корректный ответ, здесь не только посылают в гугл, но отвечают проверяя свои зания на прочность.
сам то гуглил ? первые две линки
Аноним 05/11/15 Чтв 00:25:56 #534 №225993 
14466723566240.jpg
>>225988
>котопульта
Аноним 05/11/15 Чтв 00:33:34 #535 №225994 
>>225993
Твой пертроболос - конечен
Аноним 05/11/15 Чтв 01:20:04 #536 №226009 
14466756049160.jpg
>>223277 (OP)
Что за хуйня светит в небе (ДС1) с востока от луны? Раньше этой хуйни не было в такое время (5, 6:00 МСК).

Меланхолия? Нибиру? Сириус? Вега? Что за хуйня?
Заранее спасибо.
Аноним 05/11/15 Чтв 03:27:43 #537 №226027 
>>226009
Вы заебали. Венера и Юпитер.
гугли Парад планет.
Аноним 05/11/15 Чтв 07:55:37 #538 №226065 
>>225900
Давай разберем по частям...
Что это за волшебный радиус тайм шифта? Математическую модель давай. Скорость вращения планеты какая? Околосветовая однозначно. Только вот момент, орбита корабля получается маленько длиннее, но при этом он не отстает от планеты. Он быстрее "с" летает? Ну и нумеро трез. Корабль вращается по орбите ЧД, однако относительно планеты он находится в состоянии покоя, но не орбитального движения. Почему гравитация планеты не действует на корабль.
Понарисовали блять схемок с воронками, и не понимают, что 23 года за час - пиздец какая релятивистика, люди в таких условиях не существуют.
Аноним 05/11/15 Чтв 08:06:05 #539 №226070 
>>225909
Ты сам-то эти 300 страниц читал? Распизделся тут. Расписано там все в духе самого примитивного научпопа для тупого кинотеатробыдла. А на фундаментальные косяки закрыты глаза, тип жертва во имя популяризации науки. Теоретически научным можно назвать и мир Толкиена и Мартина и все-все безумные фантазии самых ебанутых людей. Потому что типа за границами метагалактики может быть все что угодно и другие физические законы, а М-теория с 11-ю измерениями допускает существование бесконечного множества вселенных, поэтому математически в какой-нибудь из них таки есть мартышка, напечатавшая Гамлета. И это равноценно всей науке Интернолана, потому что внезапное появление кротовой норы возле Сатурна, открытой людьми из очень далекого будущего есть точно такой же рояль, основанный на тех же принципах многомерности.
Аноним 05/11/15 Чтв 08:09:01 #540 №226071 
>>225992
А ты ссылки-то читал? Пиздец, первая - очередное мусорный сайт с говном, а статья в вики просто поясняет за идеи и историю ошибок в наблюдениях.
Аноним 05/11/15 Чтв 08:20:36 #541 №226072 
>>226070
Да, читал.
>Расписано там все
Но ты ведь не читал, откуда ты знаешь?
>для тупого кинотеатробыдла
Все, кто смотрит художественные фильмы - быдло?
>А на фундаментальные косяки закрыты глаза
Да что ты говоришь, и на какие это?
Аноним 05/11/15 Чтв 08:28:37 #542 №226074 
>>226072
Быдло тот, кто не имея даже малейших знаний в области астрофизики, бежит на ресурсы, подобные этой доске и с яростью защищает понравившийся высер, прикрываясь громкими именами, не понимая даже вероятных мотивов носителей этих имен.
>Да что ты говоришь, и на какие это?
Вот в этом посте>>226065

Аноним 05/11/15 Чтв 09:15:18 #543 №226080 
Не пойму, ну почему радиус сферы (да и сфера не нужна, колечко со стабилизаторами куда лучше) обязательно делать 1 а.е., все равно о нормальной атмосферы не может быть и речи? Зависимость по площади поверхности квадратичная, любое снижение радиуса будет экономить овердохуя материалов.
Аноним 05/11/15 Чтв 09:23:25 #544 №226081 
>>226080
> Зависимость по площади поверхности квадратичная, любое снижение радиуса будет экономить овердохуя материалов.
Потому что если использовать сферу только для получения энергии и оставить обитаемую планету за ней, то:
- На планете будет как-то темновато
- Жизненное пространство не расширится
- С утилизацией всей этой прорвы энергии на одной сраной планете будут определенные проблемы

А если сделать обитаемую сферу слишком малого радиуса, там будет слегка жарковато.
Аноним 05/11/15 Чтв 09:39:04 #545 №226082 
>>226081
Количество энергии мы будем получать то же самое, но на меньшую площадь, это уже вопрос эффективности теплоотвода и утилизации этой самой энергии.
Аноним 05/11/15 Чтв 10:01:18 #546 №226083 
>>226082
Так он, вроде, и не писал про изменение кол-ва энергии.
Аноним 05/11/15 Чтв 10:02:08 #547 №226084 
>>226083
>А если сделать обитаемую сферу слишком малого радиуса, там будет слегка жарковато.
Аноним 05/11/15 Чтв 10:18:04 #548 №226085 
>>226084
Ну вот вышел постхуман на улицу, на солнышко поглазеть, яйца почесать, и изжарился нахуй.
Я как раз про удельное количество энергии, которое совсем не безразлично, если пытаться в/на этой сфере жить.
Аноним 05/11/15 Чтв 10:32:48 #549 №226086 
>>226085
Ну я не предлагаю сделать сферу/кольцо/ракушку/цилиндр/монокль совсем мелкими при нашем светиле. Нужна зависимость удельного кол-ва энергии на квадратный метр от удаленности, плюс зависимость затрат по материалам на нашу астроинженерию от радиуса и найти золотую середину.
А вообще, Миша, все хуйня, давай по-новой.
Аноним 05/11/15 Чтв 10:49:49 #550 №226087 
>>226086
Надо исходить из возможностей и материалов. А пока нет ни того, ни этого. Опять же вопрос площадей занятых под что-то еще кроме получения энергии, например если овердохуя жителей и они не питаются электричеством - надо производство пищи. Вон на кольце нивена вообще электростанции летали выше кольца, заодно формируя день и ночь, а кольцо было только комфортным жилищем.
Аноним 05/11/15 Чтв 10:52:56 #551 №226088 
>>226087
Да я не спорю, что это та еще инженерная работа для хрен знает какого века и все наши обсуждения сейчас это фантазии на тему сайфая, просто тема интересная, а в /sf/ треды полумертвые.
Аноним 05/11/15 Чтв 13:12:51 #552 №226104 
>>226086
>Нужна зависимость удельного кол-ва энергии на квадратный метр от удаленности
1.5 кВт х ( 1 а.е. / R ) ^ 2
>зависимость затрат по материалам на нашу астроинженерию от радиуса
M х ( R / 1 а.е. ) ^ 2

>>226087
>Надо исходить из возможностей и материалов. А пока нет ни того, ни этого.
и того и другого хватает.

>они не питаются электричеством - надо производство пищи.
считай 1 Га на человека этого вполне достаточно, на любой рацион, с учетом отсутвия сезонов

>>226088
>треды полумертвые.
все кто хотел обсудили, в той степени в которой могли, но это не значит что я не слежу за ними.

но вы продолжайте я зоонаблюдаю
Аноним 05/11/15 Чтв 17:55:21 #553 №226139 
>>225431
Венера и Юпитер. Марс, тусклый, да. Сатурн сейчас вообще только днем должно быть видно.

Мимо-астроном-из-телескопо-треда
Аноним 05/11/15 Чтв 23:23:37 #554 №226223 
>>224603
Всё юзали, только молча. В Химках столько народа облысело разок из-за тестов ракетного топлива, всё замалчивали и пруфов не сыскать. Я прост знаком с человеком принимавшим участие в разработке, пожилой уже естественно. Олсо, в подмосковье есть дачный посёлок где почти все имеют то или иное отношение к космической деятельности, дачки им там давали. Надо будет в следующий раз побольше кулсторей там навыхватывать и принести вам.
Аноним 05/11/15 Чтв 23:43:00 #555 №226239 
>>226223
>облысело
>из-за тестов ракетного топлива
Они им голову мыли?
Аноним 06/11/15 Птн 00:19:20 #556 №226253 
>>226239
Внутрь употребляли, ватники же
Аноним 06/11/15 Птн 00:37:09 #557 №226258 
>>225251
а маска с трубкой нужны?
Аноним 06/11/15 Птн 01:00:30 #558 №226264 
14467608304830.png
>>226258
>маска с трубкой нужны?
Вообще-то на МКС все так и летают, если ты не знал. С кем я на одной борде сижу, ужас.
Аноним 06/11/15 Птн 02:00:56 #559 №226276 
14467644563010.jpg
>>225251
алтфак в треди, все в мурзилку

там же что-то про дельфинов было
Аноним 06/11/15 Птн 02:38:24 #560 №226280 
>>226264
верхний лул
Аноним 06/11/15 Птн 22:17:06 #561 №226444 
Когда Марс заселять начнем? Даты известны?
Аноним 06/11/15 Птн 22:48:17 #562 №226447 
>>226444
Не при твоей жизни. Нет никаких дат. Может (скорее всего) и не начнём, откуда у тебя уверенность такая?
Аноним 06/11/15 Птн 23:08:08 #563 №226450 
>>226447
Земле то все равно пиздарки, если не астероид накроет, то уже через 500 млн лет Земля станет второй Венерой с кислотными дождями и температурой 500 градусов.
Так что уже сейчас нужно думать куда съябывать со страной земляшки.
Аноним 06/11/15 Птн 23:24:12 #564 №226453 
14468414524060.jpg
У меня настолько тупые вопросы, что я даже не знаю, сюда ли мне. Насколько важны сейчас для транспорта спутники GPS\ГЛОНАСС и Гугл Карты? Смогло бы человечество на данном этапе выжить и не сильно просесть в уровне жизни без них? Как люди ориентировались до них? Я вот, ребенок XXI века, не могу представить, как это - ориентироваться только по звездам, компасу и карте, каким-то непонятным хуем составленной. На суше туда-сюда еще, но когда ты посреди океана? Ебаный пиздец же? Или нет? Наверняка последние ошибки в современных картах исправлялись уже в конце XX в., на этапе получения изображений со спутников, иначе просто не представляю, как можно достаточно точно проверить карту. В общем, может ли существовать технологически развитая цивилизация без орбитальных спутников?
Аноним 06/11/15 Птн 23:26:32 #565 №226454 
>>226450
>Земле то все равно пиздарки, если не астероид накроет, то уже через 500 млн лет Земля станет второй Венерой с кислотными дождями и температурой 500 градусов.
Ну а в Марс-то уж конечно ни один астероид не въебется. И условия там прекрасные, и будут только улучшаться.
Аноним 06/11/15 Птн 23:26:51 #566 №226455 
>>226450
Это уже другой вопрос.
Но Марс не панацея от апокалипсиса, ёпта, с таким же успехом можно жить в герметичных модулях на Луне... или даже на Земле.
Тем более, если человечество протянет ещё пару миллионов лет, я даже боюсь представить, каких технологических высот оно достигнет - наверняка с климатом и мимоастероидами как-нибудь справится. А не сможет - пизда нам, щито поделать.
И даже технологически продвинутому человечеству будущего будет проще настроить жилых космических станций, чем как-то приспособить негостеприимный Марс для утренних пробежек без скафандра.
Аноним 06/11/15 Птн 23:28:01 #567 №226456 
>>226453
> Смогло бы человечество на данном этапе выжить и не сильно просесть в уровне жизни без них?
Да, смогло бы.Еще 30 лет назад не было и в помине никаких ЖПС, и ничего, нормально жили, навигировались себе по радиомаякам, звездам и картам.
Аноним 06/11/15 Птн 23:39:36 #568 №226462 
>>226456
>Еще 30 лет назад не было и в помине никаких ЖПС, и ничего, нормально жили, навигировались себе по радиомаякам, звездам и картам.
Интересно, на какой бы ты день повесился в КПЗ в 1985-м году?
Аноним 06/11/15 Птн 23:46:55 #569 №226466 
14468428151610.gif
>>226462
Аноним 07/11/15 Суб 00:44:14 #570 №226483 
>>226454
> и будут только улучшаться.
Это из-за яблонь
Аноним 07/11/15 Суб 02:57:06 #571 №226500 
Почему не делают пусковую петлю? Охуенно же придумано. Можно будет дёшево вывести всю промышленность, энергетику, научное оборудование, наладить разработку астероидов, запилить атомные двигатели. Да и 5 миллиардов чурок за 101000й километр вывезти.
Аноним 07/11/15 Суб 03:04:46 #572 №226502 
>>226500
У нас тут относительно свежий тред с перебрасыванием какашками на эту тему есть:
https://2ch.hk/spc/res/220172.html
Аноним 07/11/15 Суб 03:05:43 #573 №226504 
>>226500
>Можно будет дёшево вывести всю промышленность
Ее и сейчас всю можно недорого вывезти. По железной дороге.

>энергетику, научное оборудование, наладить разработку астероидов, запилить атомные двигатели.
Ну разумеется можно. Вставай, смена на заводе. Зарплату - карточками на продукты.

>Да и 5 миллиардов чурок за 101000й километр вывезти
Соблюдай меры предосторожности, это важно: можешь пострадать. Каждый второй был зарезан, а каждый (опять же) второй был взорван.
Аноним 07/11/15 Суб 03:25:05 #574 №226509 
>>226504
> Ее и сейчас всю можно недорого вывезти. По железной дороге.
Ну продукцию всё равно придётся в космос вывозить. Да и масштабы на Земляшке не те, поклейку звездолётов или производство антиматерии никак не наладить. А в космосе и места, и ресурсов, и энергии дохуя.
> Ну разумеется можно. Вставай, смена на заводе. Зарплату - карточками на продукты.
Если у руля будут нормальные люди, а не жлобьё, сделают по-человечески.

Аноним 07/11/15 Суб 03:40:48 #575 №226511 
>>226509
>придётся
Зачем?

>Если у руля будут нормальные люди, а не жлобьё, сделают по-человечески.
"У руля" условно твой папахер. Ну или тетя Зина, она захозом будет в ЖЭКе. А дядя Василий - мэром. Ты думаешь, чиновничий аппарат завозят с Марса? Думаешь, никогда не воспользуешься родственными связями/взяткой/просто не станешь подхалимничать? Станешь, еще и очередь растолкаешь. Один ты штоле такой...
Аноним 07/11/15 Суб 04:23:03 #576 №226519 
>>226511
Не, ну начальники мегакорпораций нормальные люди вроде, да и политики некоторые тоже. Не у всех же менталитет поднявшегося в 90х жлоба.
> Зачем?
Это круто, перспективно же. Многим людям присуще относительно бескорыстное стремление к развитию, расширению. В конце концов пора преодолевать парадигму "лень и войны - двигатель прогресса", с этой хуйнёй только всю планету засрали и расплодились как мухи.


Аноним 07/11/15 Суб 04:38:25 #577 №226523 
>>226519
>Это круто, перспективно же. Многим людям присуще относительно бескорыстное стремление к развитию, расширению.
Лыко-мочало. Много бескорыстных пусков было в последние 30 лет?
Аноним 07/11/15 Суб 10:42:33 #578 №226543 
>>226453
>как это - ориентироваться только по звездам, компасу и карте, каким-то непонятным хуем составленной
Поступай в профильный вуз, если тебе это действительно интересно. Все расскажут и покажут, если пары проебывать не будешь. А на пальцах не объяснить, все таки навигация - отдельная дисциплина.
мимогеодезист
Аноним 07/11/15 Суб 10:45:13 #579 №226545 
>>226519
>Многим людям присуще относительно бескорыстное стремление к развитию
Хиккан, плз. Ты не то, что людей не знаешь, ты себя то толком не знаешь, потому что наверняка записываешь себя в группу бескорыстных альтруистов.
Аноним 07/11/15 Суб 11:24:49 #580 №226552 
>>226545
Двачую. Надо не бескорыстное, а целенаправленное развитие, чтобы и зарплаты там были и рабочие места, и престиж. Как преподаватель в университете, в какой-нибудь США и у нас, без денег нахуй престиж и альтруизм не нужен. А с деньгами - почему бы и нет, даже если заплатят не так много, как в "Дверные ручки инк".
Аноним 07/11/15 Суб 17:04:36 #581 №226603 
Что внутри Марса? Ученые уже прокапали вглубь? На сколько глубоко?
Аноним 07/11/15 Суб 17:11:07 #582 №226605 
>>226603
Лет через сто узнают.
Аноним 07/11/15 Суб 17:59:08 #583 №226611 
>>226603
Следующий ровер будет оснащен георадаром.
Аноним 07/11/15 Суб 18:55:22 #584 №226618 
Какие существуют паранучные или лженаучные теории о космосе? Нужны примеры, анон
Аноним 07/11/15 Суб 19:34:14 #585 №226621 
>>226618
Эфирная например
https://2ch.hk/spc/res/223626.html
Аноним 07/11/15 Суб 19:47:45 #586 №226623 
14469148656530.jpg
>>226618
Плоская земля.
Аноним 07/11/15 Суб 20:19:51 #587 №226624 
14469167915830.jpg
>>226611
соус плз
Аноним 07/11/15 Суб 21:36:37 #588 №226644 
>>226603
Марс внутри полый, там живут марсиане и насмехаются над глупыми земляшками, которым приходится жить на поверхностях планет.
Аноним 07/11/15 Суб 22:42:56 #589 №226646 
Всем привет.

У меня в последнее время выработалась одна гипотеза по поводу будущего человечества.

Я считаю, что в ближайшем будущем(~50 лет) люди найдут адекватную замену углеводородам, что будет очень дешевой энергией. Ну и потом люди активно возьмутся за колонизацию космоса.

Вот у меня и вопрос:
-возможна ли колонизация в принципе?
-есть ли на других планетах/спутниках ценные ресурсы, из-за которых и будет это возможно?
- если найдется дешевая замена углеводородам, то полёты в космос будут дешевле?

И вообще, посоветуйте источников по возможностям колонизации космоса, за счёт чего всё будет работать и так далее.
Аноним 07/11/15 Суб 22:57:08 #590 №226649 
>>223277 (OP)
кто помер выше всех
Аноним 08/11/15 Вск 01:25:42 #591 №226688 
>>226646
У меня выработалась гипотеза по бессмертию человека, короче я считаю, человек скоро изобретет бессмертие и лекарство от спида/рака и т.д. и будет жить вечно.
Вот у меня вопрос:
- возможно ли бессмертие в принципе?
- если возможно, будет ли с него какой-нибудь толк?
- если бессмертие будет легко получить, сколько будут стоить макароны?
И вообще, посоветуйте источников по теме бессмертия, за счет чего оно работает и так далее.
Аноним 08/11/15 Вск 02:23:50 #592 №226704 
>>226646
>Ну и потом люди активно возьмутся за колонизацию космоса.
Зачем его колонизировать?
Аноним 08/11/15 Вск 02:34:39 #593 №226710 
>>226704
прост))
Аноним 08/11/15 Вск 03:27:27 #594 №226719 
>>226688
> посоветуйте источников
http://2ch.hk/sci/res/308582.html
> сколько будут стоить макароны?
Зависит от необходимости макарон для поддержания состояния бессмертия
> будет ли с него какой-нибудь толк?
Оно само по себе - толк, и смысл к которому стоит стремиться, ящитаю.
> возможно ли бессмертие в принципе?
а вот тут нужно define "бессмертие"
потому что возможности медицины по поддержанию жизнедеятельности организма довольно быстро растут, но тебя по видимому интересует не существование в виде овоща.

нежнее, еще нежнее. не бойся комить тралей. тред именно для этого
Аноним 08/11/15 Вск 05:56:18 #595 №226723 
>>226704
Ресурсы, проблема перенаселения, шоб летать раз в год на Марс.
Аноним 08/11/15 Вск 09:27:54 #596 №226733 
14469640742250.jpg
>>226646
1. Теоретически, если бы все человеки мира решили бы сейчас строить корабль поколений, то с учетом развития и отработки технологий, лет через 50 можно было бы запилить автономный кораблик, который довез бы до альфа центавры лет за 500, так что да, в принципе колонизация возможна. Но для этого потребовался бы вклад , оцениваемый мною на вскидку в процентов 20 от общемирового ВВП ежегодно.
2. Уж какие-нибудь ценные ресурсы обязательно найдутся. Такова человеческая природа. Если высадиться на планету целиком из говна, то человеки обязательно откопают какое-нибудь особое говно, которое будет цениться выше остального говна. Но чтобы строить целесообразную экономическую модель - нет, слишком мало данных. Пока только хоть атмосферу научились опознавать у экзопланет. Уже хорошо.
3. Замена должна быть не просто дешевой, но и прорывной в вопросе эффективности. Чтобы было целесообразно заменять топливную инфраструктуру и проектировать новые ракеты и двигатели под новое топливо. Но если и перейдут на новые виды энергии, то по показателю цена/эффективность, оно оставить далеко позади углеводороды.
Аноним 08/11/15 Вск 09:37:38 #597 №226735 
>>226624
http://mars.nasa.gov/mars2020/news/whatsnew/index.cfm?FuseAction=ShowNews&NewsID=1678
Аноним 08/11/15 Вск 09:50:34 #598 №226736 
>>226649
Добровольский, Волков, Пацаев
Аноним 08/11/15 Вск 10:45:28 #599 №226737 
>>226646
>>226733
>если найдется дешевая замена углеводородам, то полёты в космос будут дешевле?
>3. Замена должна быть не просто дешевой, но и прорывной в вопросе эффективности.
Не хочу вас обоих огорчать, но топливо стоит так мало в стоимости запуска, что его можно не учитывать просто.
Керосин и кислород, сирзли? Да и потребление космосом углеводородов это ничтожные доли процента от потребления автомобилями
Аноним 08/11/15 Вск 11:25:27 #600 №226740 
14469711273330.png
Чего сложного пустить ракету на порохе до космоса? Китайцы пилят крутые йобы, которые похожи на спайс кербал программ, а не пускают ракеты в космос. Понятно когда кусок мяса запускают, но чего сложно без орбиты пустить какую-нибудь хуйню, ну?
Аноним 08/11/15 Вск 11:30:54 #601 №226741 
>>226740
Можно, и пускали. У некоторых даже на второй-третьей ступени после ЖРД стояло.
Аноним 08/11/15 Вск 11:31:45 #602 №226742 
>>226740
Если под порохом понимать твердотопливные двигатели в целом (именно на порохе - это слишком толсто даже для этого треда), то:
1. Низкая эффективность топлива.
2. Включив ты уже не выключишь и не сбавишь газок (впрочем тротлинг на жрд тоже не такой уж частый гость).
3. Формула Циолковского и прочие дельты какбы намекают, что неплохо было бы полезный груз вывести, при этом не стартуя слишком большими йобами. В этом плане жрд интереснее.
Так обычно бустеры твердотоплиные, на старте пинка дать.
Аноним 08/11/15 Вск 11:39:32 #603 №226744 
>>226742
>именно на порохе - это слишком толсто даже для этого треда
Поясни плз дауну, почему толсто? То, что он взорвется? Так разве нельзя его как-нибудь спрессовать по особому, или еще какую хуйню сделать? Или порох как топливо - слишком неэффективный?
Аноним 08/11/15 Вск 11:52:04 #604 №226746 
>>226744
Ты все тот же даун, сто писал про стоимость углеводородов? В стоимости запуска цена топлива очень малая величина, так что топливо надо брать наиболее эффективное. Да и ты что думаешь, ты сделаешь картонную ракету, набьешь порохом и ХУЯК! Она сама вышла на НОО с заданными параметрами, ура товарищи?
Аноним 08/11/15 Вск 11:55:38 #605 №226748 
>>226737
Так я о чем и говорю. Если сейчас изобретут йоба-лигроин, или какой-нибудь основанную на каком-нибудь йоба-принципе квантового обмена электронов смесь инертных газов или прочую фантастику, цена которых будет в сотню-другую раз меньше современных вариантов, но потребует замены инфраструктуры, замены оборудования и двигла, то администраторы еще почешут репы, а нужна ли вообще такой ценой эта замена, пусть и экономичная в долгой перспективе.
Аноним 08/11/15 Вск 11:57:04 #606 №226749 
>>226746
Нет, я другой даун. Так причина - в низкой эффективности пороха?
Аноним 08/11/15 Вск 12:01:22 #607 №226750 
>>226749
Ну попробую иначе. Вот перед тобой мопед дедушки ездит даже на самогоне, правда тормозов нет, и на полной скорости бывает отваливается что-нибудь и рено логан, на чем ты поедешь в путешествие 10 000 км, с чемоданом который надо дяде отвезти (это цель), весом 20кг, стоимостью 200 т.р. на логане или мопеде, но мопед более неприхотлив у тебя к топливу.
Аноним 08/11/15 Вск 12:09:51 #608 №226751 
>>226750
Да все уже поняли про стоимость топлива. Спрашиваю про твердое топливо же. Твердотопливное двигло ведь существует и используется? Чем же настолько плох порох по сравнению с другими твердыми горючими веществами, что
>именно на порохе - это слишком толсто даже для этого треда
Аноним 08/11/15 Вск 12:24:50 #609 №226753 
>>226751
Да блядь, я уже не про стоимость топлива даже, ты выводить что-то хочешь или фейрверки попускать, только очень латентно?
Аноним 08/11/15 Вск 12:41:13 #610 №226754 
>>226753
> > Почему порох плохое топливо?
> Порох плохое топливо.
Да объясни ты нормально уже человеку.
Аноним 08/11/15 Вск 12:42:38 #611 №226755 
>>226753
Ты ответить можешь нормально? Он взрывается, так? Или, он взрывается и годится только для петард (крайне неэффективен)? Или что-то еще?
Аноним 08/11/15 Вск 13:10:35 #612 №226759 
>>226754
>>226755
Ну ок, даже не знаю с чего начинать... Он имеет маленький удельный импульс, горит неравномерно, гигроскопичен (сейчас сущестуют гораздо более эффективные твердые топлива), в целом у твердотопливного двигла самого есть проблемы, невозможно регенеративное охлаждение сопла, при том что оно быстрее выгорает, обнуление тяги возможно только взрывом стенок, чтобы пламя шло в стороны, а не из сопла, в полете ничего поменять нельза, все характеристики закладываются на этапе проектировки (создание профиля тяги). Ну и просто вывод это нечто большее, чем запуск шутихи, там электроника, нужна некая управляемость, эффективность, что трд не дает. Поэтому в основном на старте дает пинок, а дальше все жрд делает.
вот, держи, почитай. Там еще серия про различные двигатели и топлива у него http://geektimes.ru/post/209242/
Аноним 08/11/15 Вск 13:16:55 #613 №226760 
что за планета и кто такой карл саган на оп-пике?
Аноним 08/11/15 Вск 13:36:57 #614 №226763 
>>226760
Плутон. Сагана загуглить лень, да? Нигга - Нил Тайсон.
Пиши с большой буквы, брат наш меньший.
Аноним 08/11/15 Вск 13:44:35 #615 №226765 
>>226763
В чем заключается тонкость этой пикчи?
Аноним 08/11/15 Вск 14:02:49 #616 №226767 
>>226763
>http://hi-news.ru/space/nil-de-grass-tajson-obyasnil-pochemu-nam-ne-stoit-boyatsya-kosmosa.html
>В фильме «Гравитация» я насчитал около 10 ляпов в надежде на то, что их оправдают, но нет. Например, я не понял, почему Сандра Буллок, доктор по призванию, ремонтирует космический телескоп Хаббла. Уберите ее от моего телескопа!
ля чето у меня в башке аж подвинулось, после того как я подумал что ктото может назвать Хаббл - Мой телескоп!
Аноним 08/11/15 Вск 14:30:19 #617 №226770 
>>226765
В том, что Тайсон - популяризатор науки и Саган для него пример, о чем он неоднократно говорил. Тайсон один из тех, кто приложил руку к тому, что бы Плутон пидорнули из планет в карликовые планеты.
Аноним 08/11/15 Вск 14:32:36 #618 №226771 
>>226767
Съеби с моего двача!
Аноним 08/11/15 Вск 15:33:29 #619 №226795 
Почему бы не отправить в сторону потенциально обитаемых планет зонды с мощными источниками радиационного излучения? Тогда бы достаточно развитая цивилизация могла обнаружить их по фону и установить контакт с нами.
Аноним 08/11/15 Вск 15:37:58 #620 №226796 
14469862787250.jpg
>>226795
>в сторону потенциально обитаемых планет
Список в студию.
Аноним 08/11/15 Вск 15:39:27 #621 №226797 
>>226796
Глизе чето-там
Аноним 08/11/15 Вск 15:54:31 #622 №226806 DELETED
>>226797
Вся суть теперяшнего Спейчача. Хочу вход по паспорту, и нотариально заверенному скриншоту о непосещении /вм/.
Аноним 08/11/15 Вск 16:10:11 #623 №226813 DELETED
>>226806
> Хочу вход по паспорту

Тот, кто жертвует анонимностью ради адекватности, не достоин ни того, ни другого. И рано или поздно потеряет и то и другое.
Аноним 08/11/15 Вск 16:15:10 #624 №226814 
>>226796
https://ru.wikipedia.org/wiki/%D0%A1%D0%BF%D0%B8%D1%81%D0%BE%D0%BA_%D0%BF%D0%BE%D1%82%D0%B5%D0%BD%D1%86%D0%B8%D0%B0%D0%BB%D1%8C%D0%BD%D0%BE_%D0%B6%D0%B8%D0%B7%D0%BD%D0%B5%D0%BF%D1%80%D0%B8%D0%B3%D0%BE%D0%B4%D0%BD%D1%8B%D1%85_%D1%8D%D0%BA%D0%B7%D0%BE%D0%BF%D0%BB%D0%B0%D0%BD%D0%B5%D1%82

https://ru.wikipedia.org/wiki/%D0%A1%D0%BF%D0%B8%D1%81%D0%BE%D0%BA_%D1%8D%D0%BA%D0%B7%D0%BE%D0%BF%D0%BB%D0%B0%D0%BD%D0%B5%D1%82_%D0%B2_%D0%BE%D0%B1%D0%B8%D1%82%D0%B0%D0%B5%D0%BC%D0%BE%D0%B9_%D0%B7%D0%BE%D0%BD%D0%B5
Аноним 08/11/15 Вск 16:20:23 #625 №226815 
>>226795
Более мощный, чем звезда, вокруг которой вращается их планета?
А ты смешной.
Аноним 08/11/15 Вск 16:20:44 #626 №226816 
>>226795
Планета на которой живут ебланы, демаскирующие свой хоуволд сами, заслуживает разрушения.
Аноним 08/11/15 Вск 16:21:25 #627 №226817 
>>226816
*homeworld
Возраст бытия, епт [Анон] Анон 08/11/15 Вск 16:31:21 #628 №226822 
Народ, я в астрономии нуб, но тут недавно, читая всякие там факты о космосе, наткнулся на взаимоисключающие.

Возраст вселенной = 14 миллиардов лет.
Граница наблюдаемо вселенной = 90 миллиардов световых лет.

Может, я чего-то не понимаю(скорее всего, так и есть), но разве свету не надо 90млд лет на прогулку от краешка наблюдаемой до очей наших? То есть получается, что Аллах таки запилил землю 90 миллиардов лет назад? Ничего не понимаю. Просветите.
Аноним 08/11/15 Вск 16:42:40 #629 №226824 
>>226822
> Граница наблюдаемо вселенной = 90 миллиардов световых лет.
Я не знаю, откуда ты это вытащил, но лучше засунь этот бред обратно.
Аноним 08/11/15 Вск 16:43:56 #630 №226825 
>>226822
Я надеюсь ты можешь в английский.
https://www.quora.com/How-can-it-be-understood-that-the-universe-is-93-billion-light-years-across-and-yet-only-13-8-billion-years-old-1
Аноним 08/11/15 Вск 16:48:17 #631 №226826 
>>226822
Ты путаешь "горизонт событий" и "горизонт частиц". Твои "90 млрд." - это где они сейчас реально находятся, из-за растяжения пространства. Твои "14 млрд." - это место, где они были испущены.

Для нас, на таких расстояниях, важна лишь меньшая цифра. Это информация. То, что на "90 млрд.", ты увидеть не можешь: скрыто горизонтом Метагалактики.
Аноним 08/11/15 Вск 16:54:35 #632 №226828 
Какова температура в космическом простраснтве? По цельсию.
Аноним 08/11/15 Вск 16:59:09 #633 №226830 
>>226828
-270.45
Аноним 08/11/15 Вск 17:00:19 #634 №226831 
>>226828
Смотря как мерить и где. Если повесить на низкую околосолнечную орбиту градусник, он расплавится нахуй, потому что излучение от Солнца. Если его же повесить где-нибудь подальше между галактиками - покажет примерно -270 - температуру реликтового излучения.
Аноним 08/11/15 Вск 17:01:40 #635 №226832 
>>226828
>Какова температура в космическом простраснтве? По цельсию.
В отсутствии космических тел (планет, звезд, говна и палок; хотя они (хоть и далеко, но) всегда есть - температуре реликта:
https://ru.wikipedia.org/wiki/Реликтовое_излучение

По цельсию - переведешь в гугле. Типа ёпта -270.
Аноним 08/11/15 Вск 17:02:21 #636 №226833 
>>226830
>>226831
>>226832
Что будет с человеком если он внезапно окажется в космическом пространстве без соответствующего снаряжения? Он превратится в кусок льда? И как быстро?
Аноним 08/11/15 Вск 17:04:16 #637 №226834 
>>226833
Лечу по космическому пространству в майке и старых труханах. Задавайте свои ответы.
Аноним 08/11/15 Вск 17:10:11 #638 №226835 
>>226833
>Что будет с человеком если он внезапно окажется в космическом пространстве без соответствующего снаряжения? Он превратится в кусок льда? И как быстро?
Заебал. Прошлый тред еще не потонул.
>>220339
Аноним 08/11/15 Вск 17:59:12 #639 №226843 DELETED
>>226813
достойно, кровь патриотов должна проливаться.
Аноним 08/11/15 Вск 18:18:51 #640 №226847 
>>223277 (OP)
Посмотрел к краю вселенной и не понял как они измеряли, они тупо летели в одну стороны на n световых лет, либо уменьшали масштаб на n световых лет или что? Запутался там на пульсахар и прочем
Аноним 08/11/15 Вск 20:14:52 #641 №226866 
>>226847
>Посмотрел к краю вселенной
В существующей научной парадигме это невозможно.
Аноним 08/11/15 Вск 21:22:14 #642 №226876 
>>223529
А на сколько остальные места горячие?
И почему они горячие?
Там же пустота, чему там проводить тепло?
Уже прыгаю в биореактор, не обоссывайте
Аноним 08/11/15 Вск 21:38:22 #643 №226878 
14470079025120.png
>>226876
Аноним 08/11/15 Вск 23:35:50 #644 №226889 
>>226878
Чому когда рисуют наблюдаемую вселенную, то её рисуют в виде эллипса, а не круга?
Аноним 08/11/15 Вск 23:51:11 #645 №226890 
14470158715710.jpg
>>226889
Потому что не очень удобно проецировать сферу на круг. А так - конкретно на пике, достаточно распространенная проекция Моллвейде.
Аноним 08/11/15 Вск 23:53:55 #646 №226891 
14470160351210.jpg
14470160351241.jpg
14470160351252.jpg
>>226889
Потому что это сфера изнутри.
Аноним 08/11/15 Вск 23:55:13 #647 №226892 
14470161134470.jpg
>>226889
проекция такая сферы на плоскость.
аналог https://en.wikipedia.org/wiki/Winkel_tripel_projection для Земли.

Этих проекций, кстати, есть оче много вариантов. Вот можешь тут побаловаться:

http://earth.nullschool.net
Аноним 08/11/15 Вск 23:58:49 #648 №226893 
>>226892
> аналог https://en.wikipedia.org/wiki/Winkel_tripel_projection для Земли.
Это не аналог, это вполне земная проекция, использованная для космоса: https://en.wikipedia.org/wiki/Mollweide_projection В вики даже аналогичная карта в пример приведена.
Аноним 09/11/15 Пнд 00:01:42 #649 №226894 
14470165027870.jpg
>>226893
пожалуй ты прав, а я в глаза ебусь. Ну суть мы ему расписали. Алсо, я вот с этой проекции проигрываю. Зачем ее вообще сделали? В чем ее польза?
Аноним 09/11/15 Пнд 00:05:52 #650 №226895 
>>226894
Минимальные искажения.
Аноним 09/11/15 Пнд 00:07:39 #651 №226896 
>>226894
Чтобы по континентам не резать, и при этом оставить поменьше искажений. Судя по тому, что такие проекции я вижу в основном в контексте "глядите чего географы удумали" - не самая полезная идея.
Аноним 09/11/15 Пнд 00:11:43 #652 №226900 
>>226894

проекция равных площадей земной поверхности, за счёт искажения расстояний.
Аноним 09/11/15 Пнд 00:19:37 #653 №226902 
>>226889
Алсо, вот еще чуть конкретнее: http://scienceblogs.com/startswithabang/2013/09/27/ask-ethan-4-weird-astronomy-maps/ и перевод http://geektimes.ru/post/252312/
Аноним 09/11/15 Пнд 10:54:39 #654 №226986 
>>226771
не, ну за этим утверждением ничего не стоит, да и объект, ну ты понел
Аноним 09/11/15 Пнд 10:56:58 #655 №226987 
>>226896
можно напечатать и практически сходу склеить чтото похожее на глобус
Аноним 09/11/15 Пнд 11:00:39 #656 №226988 
перекатите первую пикчу, уж больно мне она стала нравиться, чтото в ней есть что предстоит еще понять
Аноним 09/11/15 Пнд 13:14:31 #657 №226993 
>>226988
Ну охуеть, а может ты просто сохранишь ее и будешь вбрасывать в тред, если тебе так нравится. Перекатите ему, совсем охуел.
Аноним 09/11/15 Пнд 15:45:24 #658 №227016 
>>226993
ну если бы я был настолько селфишь, как говорят наши потенциальные друзья, то ...
а так я поднял вопрос, я только начал входить во вкус этой картинки, после того как доброаноны пояснили ее смысл
Аноним 09/11/15 Пнд 23:38:20 #659 №227071 
14471015004880.png
14471015004981.jpg
Что там происходит?
Аноним 09/11/15 Пнд 23:42:29 #660 №227074 
>>227071
И ещё
https://www.youtube.com/watch?v=qzXXCNhCmME
Аноним 10/11/15 Втр 01:58:07 #661 №227098 
>>227071
космический сперматозоид прилетел покорять земляшку
Аноним 10/11/15 Втр 02:04:12 #662 №227099 
14471102520920.jpg
>>227071
Вояки свои ракетки пускают. Смари, вон у меня фоточка с запуском ракеты Тополя есть.
Аноним 10/11/15 Втр 02:05:25 #663 №227100 
14471103256670.jpg
>>227099
2spooky4plebs
Аноним 10/11/15 Втр 07:29:26 #664 №227114 
14471297667000.jpg
>>227071
Вояджер возвращается домой под маскировкой
Аноним 10/11/15 Втр 09:01:10 #665 №227118 
Почему вещество/объект не может упасть на звезду/черную дыру по прямой? Я понимаю, что большинство будет затянуто притяжением по касательной, но если он изначально прямо в нее летит?
Аноним 10/11/15 Втр 09:02:40 #666 №227119 
>>227118
Потому что орбита
Аноним 10/11/15 Втр 10:39:58 #667 №227127 
>>227118
> Почему не может?
Может. Прямая - один из видов конических сечений, вырожденное правда.

Однако, в случае с ЧД все таки не может. У неё там это, эргосфера.
Аноним 10/11/15 Втр 11:23:41 #668 №227130 
>>227127
Я из этой википедии нихуя не понял, кто ее пишет вообще?
>Чтобы удержаться от падения на поверхность горизонта событий, тело, находящееся под поверхностью статического предела, должно вращаться с положительной угловой скоростью вокруг чёрной дыры (в сторону вращения чёрной дыры).
Кому должно? Зачем удерживаться? Объясните на пальцах.
Аноним 10/11/15 Втр 11:30:08 #669 №227132 
Вот еще вопрос не по хуйне: метеор, сгорающий в атмосфере, увеличивает энергию Земли или уменьшает? В плане, ближе она становится к Солнцу или дальше? Интуиция подсказывает, что напрямую зависит от направления падения, с другой стороны, до Земли он все равно не долетает, только атмосферу нагревает немного. С третьей стороны, температура атмосферы на орбиту Земли не влияет вообще.
Аноним 10/11/15 Втр 11:45:30 #670 №227143 
Посоны, что думаете о идее Илона Маска ёбнуть по полюсам термоядерными бомбами для создания атмосферы/парникового эффекта? Взлетит или хуйня? Там же магнитного поля нет, не улетит ли большая часть взлетевшего безвозвратно? Реальные ученые высказывались как-то по этому поводу?
Аноним 10/11/15 Втр 11:46:56 #671 №227144 
>>227132
Радиус орбиты зависит ещё и от массы.
Аноним 10/11/15 Втр 11:54:13 #672 №227147 
>>227144
Как зависит?
Аноним 10/11/15 Втр 12:00:21 #673 №227150 
>>227147
При одинаковой скорости, более массивный объект будет иметь больший радиус.
Аноним 10/11/15 Втр 12:08:01 #674 №227153 
>>227143
Тут весь спектр мнений по этому вопросу.
https://2ch.hk/spc/res/215183.html
Аноним 10/11/15 Втр 12:31:32 #675 №227162 
>>223277 (OP)
Зачем СССР сдался в космической гонке? Допилили бы Н1Ф или построили бы гептилку УР700 и полетели тоже воткнулись бы. А потом и Марс бы вовсю осваивали к нашему времени.
Чугуния для ракет полно, соляры завались, человеческого мяса для клепания ракет и отправки в ебеня тоже дохерища у обоих стран. Обе страны деньги печатали как хотели, нахуя сдаваться было?
Теперь сидим как сычи на НОО и нихуя не делаем, пиздос же.
Аноним 10/11/15 Втр 12:35:35 #676 №227165 
Можно потихоньку выкатываться.
https://2ch.hk/spc/res/227164.html
Аноним 10/11/15 Втр 12:35:39 #677 №227166 
>>227162
Я думаю, что марс (и другие планеты) должны осваивать не люди, а более примитивные организмы, которые по крайней мере смогут жить (и эволюционировать) в таких условиях. А с людьми как получится, технологий для комфортного переката пока не так много.
Аноним 10/11/15 Втр 12:37:14 #678 №227168 
>>227150
> При одинаковой скорости, более массивный объект будет иметь больший радиус.
Чтобы заметить эту разницу не на бумаге, на Землю придется скинуть очень много метеоритов.
Аноним 10/11/15 Втр 12:39:17 #679 №227170 
>>227168
Или один, но здоровенный.
Аноним 10/11/15 Втр 12:41:07 #680 №227171 
>>227170
Главное при этом Землю не расколоть, а то масса наоборот уменьшится.
Аноним 10/11/15 Втр 19:13:07 #681 №227272 
>>227130
>Кому должно
Законам физики.
>Зачем удерживаться?
Шоб не raspeedorseelo.
Аноним 18/11/15 Срд 02:43:22 #682 №229685 
14478038028810.jpg
А почему ГГ из интерсталера проходя через гиперпространство, не встретил вашу-чан?
Аноним 18/11/15 Срд 02:46:26 #683 №229686 
Как выглядит более подробно, планетарная система из к\ф Чёрная Дыра? Насколько возможна данная система?
comments powered by Disqus

Отзывы и предложения